PANCE Practice Questions

Ace your homework & exams now with Quizwiz!

(C) Descending colon The most common sites of obstruction are descending colon

83-year-old man is diagnosed on colonoscopy to have cancer of the colon. He refuses surgical intervention and after a 3-month follow- up period is admitted to the emergency department with large-bowel obstruction. Carcinoma of the colon is most likely to obstruct if found in the (A) Cecum (B) Ascending colon (C) Descending colon (D) Rectum (E) Transverse colon

B. G551D this patient should be evaluated for cystic fibrosis

A 1 day-old boy develops progressing abdominal distension, bilious vomiting and failure to pass a meconium stool. Abdominal radiographs show dilated loops of small bowel. Which of the following genetic mutations should the patient be evaluated for? A. ΔF508 B. G551D C. Q1412X D. R117H E. W1282X

(B) human herpes virus 6 Roseola

A 10-month-old child has a temperature of 104°F (40°C) for 4 days without other signs. On the fourth day a rose pink, maculopapular rash appears and the temperature returns to normal. What is the most likely diagnosis? (A) echovirus (B) human herpes virus 6 (C) measles virus (D) group A streptococcus (E) typhus

(E) An upper GI series it would not visualize the cyst. Ultrasound may diagnose a choledochal cyst, showing size and location, but is not always diagnostic. CAT scan and MRCP can show size, location, and extent of disease. ERCP visualizes the distal duct anatomy well, while PTC is better at visualizing the proximal ductal anatomy.

A 13-year-old female presenting with RUQ abdominal pain is suspected of having a choledochal cyst. Which of the following studies would be least helpful in confirming the diagnosis in this case? (A) Computed tomography (CT) scan (B) Percutaneous transhepatic cholangiography (C) Endoscopic retrograde cholangiopancreatography (D) Magnetic resonance cholangiopancreatography (MRCP) (E) Upper GI series

(A) Neisseria meningitidis

A 16-year-old adolescent presents with temperature of 103.1°F (39.5°C) and purpuric skin lesions. The child is found to have a blood pressure of 60/30 and heart rate of 180 with bounding pulses and a respiratory rate of 40. What infectious agent is the most likely to cause the above symptoms? (A) Neisseria meningitidis (B) Haemophilus influenzae (C) Streptococcus pneumoniae (D) Staphylococcus aureus (E) Group B beta-hemolytic streptococcus

(G) Exploratory laparotomy

A 16-year-old boy presents to the emergency department with a gunshot wound to the abdominal cavity. (A) Peritoneal lavage (B) Wound exploration (C) Sonogram (D) Paracentesis (E) CT with intravenous and oral contrast (F) IVP (G) Exploratory laparotomy (I) Angiogram

(C) azithromycin Pertussis

A 2-week-old infant presents with apnea. The infant was born at term after an uncomplicated pregnancy. The mother of this baby had rhin- orrhea and cough that started 3 weeks ago and now she has a severe persistent cough with post-tussive emesis. Which treatment should be initiated? (A) ceftriaxone (B) amoxicillin (C) azithromycin (D) vancomycin (E) amantidine

B. Adenosine 12 mg IV

A 27-year-old woman with a rapid narrow-complex regular tachycardia continues to have palpitations after vagal maneuvers and adenosine 6 mg. Pulse is 180; other vital signs are normal. She is receiving 2 L oxygen by nasal cannula. What is the next appropriate intervention? A. Adenosine 6 mg IV B. Adenosine 12 mg IV C. Metoprolol 5 mg IV D. Synchronized cardioversion

(C) Toxic megacolon

A 35-year-old man has known ulcerative colitis. Which of the following is an indication for total proctocolectomy? (A) Occasional bouts of colic and diarrhea (B) Sclerosing cholangitis (C) Toxic megacolon (D) Arthritides (E) Iron deficiency anemia

B. Emergent endoscopy with sclerotherapy

A 54-yo man with a history of "alcoholic liver disease" presents with frank hematemesis, a blood pressure of 80/40, pulse rate of 110, and respiratory rate of 26. After assessing and managing the airway, which of the following is most likely to provide definitive treatment? A. Blood products B. Emergent endoscopy with sclerotherapy C. Normal saline 1 L bolus IV D. Sengstaken-Blakemore tube E. Vasopressin

d) Hyperkalemia

A 63-year-old man is seen 3-days post-op and shows signs of fever, abdominal pain, nausea, and anorexia. His urine output is 100 ml over the last 24 hrs. His blood pressure is 84/62, and his pulse is 138. His response to this physiologic state includes which of the following? a) Increase in sodium and water excretion b) Increase in renal perfusion c) Decrease in cortisol levels d) Hyperkalemia e) Hypoglycemia

(D) GIST

A 63-year-old woman is admitted to the hospital with a UGI bleed that subsides spontaneously within a short time after admission. A barium study shows a gastric ulceration that is described by the radiologist as having a "doughnut sign." What is the most likely diagnosis? (A) Lipoma (B) Gastric ulcer (C) Ectopic pancreas (D) GIST (E) Carcinoma

(E) 24-36 months

A toddler has started toilet training. What is the typical age of this child? (A) 4-6 months (B) 6-9 months (C) 15-18 months (D) 18-24 months (E) 24-36 months

C. 34 to 38 cm

A women is being evaluated at her 36-wk OB appointment. She is not obese, her bladder is empty, and she does not have any complications. The fetus is in a cephalic position by Leopold maneuver. You measure her fundal height. What should it measure? A. 32 to 34 cm B. 32 to 36 cm C. 34 to 38 cm D. 35 to 39 cm E. 32 to 39 cm

(C) 15-18 months

An infant is able to self-feed. What is the usual age? (A) 4-6 months (B) 6-9 months (C) 15-18 months (D) 18-24 months (E) 24-36 months

(C) Fistulisation to adjacent organs such as the bladder, with insueing colovesical fistula

Complications of diverticulitis include: (A) Carcinoma of the colon (B) Extraintestinal manifestations such as arthritis, iritis, and skin rashes (C) Fistulisation to adjacent organs such as the bladder, with insueing colovesical fistula (D) Artheriovenous fistulae of the intestine (E) Sclerosing cholangitis

(C) 7 years

Concerns about childrens' TV watching are being discussed with a group of parents. The age at which a child can reliably distinguish fantasy from reality occurs at which of the following ages? (A) 4 years (B) 6 years (C) 7 years (D) 8 years (E) 10 years

D. initiate Heimlich maneuver

During a wilderness emergency medicine training course, conducted in a remote location, a member of the group begins choking on a piece of food. The patient has lost the ability to cough and seems unable to breathe.What is the next best step in management? A. blind sweep of the oral cavity B. emergent cricothyrotomy C. emergent tracheostomy D. initiate Heimlich maneuver

(C) carbamazepine

Management with multiple-dose activated charcoal may be indicated in the overdose of which of the following? (A) iron (B) cyanide (C) carbamazepine (D) tricyclicantidepressants (E) methanol

C) Endometrial

Patients with the gene for hereditary non-polyposis colorectal cancer have an increased risk for which gynecologic cancer? A) Ovarian B) Cervical C) Endometrial D) Breast E) Fallopian tube

(E) 45,X or XO (Turner syndrome)

Redundant skin at nape of neck, low posterior hairline, edema of dorsum of feet in newborn period, cardiac and renal anomalies (A) trisomy 13 (Patau syndrome) (B) trisomy 18 (Edwards syndrome) (C) 5p deletion (cri-du-chat syndrome) (D) trisomy 21 (Down syndrome) (E) 45,X or XO (Turner syndrome)

B. β-Blocking agents might improve cardiac output

Regarding treatment of heart failure in patients with diastolic rather than systolic dysfunction: A. Aggressive therapy with diuretics is more effective B. β-Blocking agents might improve cardiac output C. Both are associated with impaired cardiac contractility D. Most patients with CHF have diastolic dysfunction E. Ventricular filling pressures are higher in systolic dysfunction than in diastolic dysfunction

(B) Iliac artery

Stenosis of which of the following vessels is associated with the highest patency rates fol- lowing angioplasty or stenting? (A) Medial circumflex artery (B) Iliac artery (C) Superficial femoral artery (D) Popliteal artery (E) Tibial arteries

(A) Enterobius vermicularis infection. Pinworms

The mother of a 3-year-old child in your office for a well-child visit mentions that her child has been scratching his bottom lately. The mother reports to you that on a few occasions she has seen what appears to be rice in his diaper. Which of the following is the most likely explanation? (A) Enterobius vermicularis infection. (B) The child has malabsorption disorder. (C) The child eats rice frequently. (D) Strongyloides stercoralis infection. (E) Taenia solium infection.

(B) caution against leaving small items on the floor because infant could pick them up

The parents of a 3-month-old boy who is growing well come to the pediatrician's office for a well-child check. Which of the following guidance items regarding growth and development are appropriate to discuss at this visit? (A) may leave infant unattended on couch or changing table as long as he is not rolling over yet (B) caution against leaving small items on the floor because infant could pick them up (C) infant still needs to be fed every 2 hours at night (D) expect him to coo, smile and laugh (E) may place infant on his abdomen to sleep

(B) Peaked T waves

Twenty-four hours after colon resection, urine output in a 70-year-old man is 10 mL/h. Blood chemistry analysis reveals sodium, 138 mEq/L; potassium, 6 mEq/L; chloride, 100 mEq/L; bicarbonate, 14 mEq/L. His metabolic abnormality is characterized by which of the following? (A) Abdominal distension (B) Peaked T waves (C) Narrow QRS complex (D) Cardiac arrest in systole (E) J wave or Osborne wave

C) No adverse outcome

What is the most likely pregnancy outcome for a patient who contracts Parvovirus B19 causing erythema infectiosum (Fifth Disease) in the first trimester? A) Congenital anomalies B) Spontaneous abortion C) No adverse outcome D) Childhood developmental delays E) Severe fetal anemia

C) Previous ectopic pregnancy

What risk factor confers the highest risk for an ectopic pregnancy? A) History of PID B) Previous tubal reanastomosis C) Previous ectopic pregnancy D) Smoking E) Previous tubal ligation

D) Monochroionic/Diammniotic

What type of placentation is the most common in monozygotic twins? A) Diamniotic/Dichorionic B) Monochroionic/monoamniotic C) Dichorionic/monoamniotic D) Monochroionic/Diammniotic

(D) Common femoral artery

Which is the most common site at which an arterial embolus lodges? (A) Aortic bifurcation (B) Popliteal artery (C) Tibial arteries (D) Common femoral artery (E) Iliac artery

B. Brushfield spots Epstein pearls: small white cysts along the median raphe of the hard palate Brushfield spots: small white spots on the periphery of the iris, in Down's syndrome subgaleal hemorrhage: bleeding in the potential space between the skull periosteum and the scalp galea aponeurosis. Adamson's fringe: location of the terminal tuft of the hyphae (in tinea capitis)

Which of the following can be found in the eyes during the physical examination of a newborn? A. Epstein pearls B. Brushfield spots C. Subgaleal hemorrhages D. Adamson's fringe

C. inadequate dietary protein Low dietary Calcium, not protein, is a risk factor for osteoporosis. All of the others are risk factors.

Which of the following is NOT a risk factor for the development of osteoporosis? A. low testosterone levels in men B. low levels of physical activity C. inadequate dietary protein D. cigarette smoking E. chronic corticosteroid use

(C) meconium ileus

You are a practicing pediatrician in a state where cystic fibrosis is not a routine part of newborn screening. Which of the following symptoms in a newborn infant would prompt you to test for cystic fibrosis? (A) pneumonia (B) intrauterine growth retardation (C) meconium ileus (D) wheezing (E) hypochloremic alkalosis

(D) Fever

You're evaluating a 4-month-old infant who presents with a fever of 39.4°C (103°F) and tonic-clonic seizure. There's no prior history of seizures or epilepsy. A lumbar puncture is unrevealing. Which one of the following is the likely cause of this infant's seizures? (A) Meningitis (B) Encephalitis (C) Malignancy (D) Fever (E) Epilepsy

D) IV estrogen

A 13 yo G0 presents to the ER several months after menarche with acute, heavy vaginal bleeding. She has a diagnosis of type 2 diabetes. On H&P, you also note hirsuitism and a BMI of 40. She also has a sister with von Willebrand disease. The best initial treatment is: A) Hysteroscopy B) D&C C) Desmopressin D) IV estrogen E) Progesterone

(C) CT of the abdomen

A 22-year-old woman presents to the emergency department with a chief complaint of severe left upper quadrant (LUQ) pain after being punched by her husband. Her blood pressure is 110/70 mm Hg, pulse is 100 bpm, and respiration rate is 24 breaths per minute. The best means to establish a diagnosis is which of the following? (A) FAST (B) Physical examination (C) CT of the abdomen (D) Peritoneal lavage (E) Upper gastrointestinal (GI) series

(B) Tension pneumothorax

A 26-year-old man is stabbed in the right intercostal space in the midclavicular line and presents to the emergency department. On examination, subcutaneous emphysema of the right chest wall, absent breath sounds, and a tra- chea shifted to the left are noted. What is the most likely serious diagnosis? (A) Pneumothorax (B) Tension pneumothorax (C) Massive hemothorax (D) Hemopneumothorax (E) Chest wall laceration

(D) Streptococcus pneumoniae

A 4 -year-old child presents to your office with fever, and increased work of breathing manifested by tachypnea and retractions. A chest x-ray confirms lobar pneumonia. What is the most likely etiology of pneumonia in this child? (A) Mycoplasma pneumoniae (B) Streptococcus pyogenes (C) Chlamydophila pneumoniae (D) Streptococcus pneumoniae (E) Staphylococcus epidermidis

(E) Hepatic adenoma

A 40-year-old female alcoholic is suspected of having a hepatic mass. Percutaneous ultrasound- guided liver biopsy is contraindicated in which of the following? (A) Hepatocellular carcinoma (B) Metastatic carcinoma (B) Cirrhosis (D) Hepatitis C (E) Hepatic adenoma

A) Toxic megacolon in ulcerative colitis

A 40-year-old man with a long history of bloody diarrhea presents with increased abdominal pain, vomiting, and fever. On examination, he is found to be dehydrated and shows tachycardia and hypotension. The abdomen is markedly tender with guarding and rigidity. What is the most likely cause? (A) Toxic megacolon in ulcerative colitis (B) Small-bowel perforation from regional enteritis (C) Perforated carcinoma of the sigmoid colon (D) Volvulus of the sigmoid colon (E) Acute perforated diverticulitis

(D) 10-20 years

A 54-year-old man with diarrhea is found to have ulcerative colitis. Colectomy should be advised in patients with ulcerative colitis who have symptoms that persist for more than which of the following? (A) 1 month (B) 6 months (C) 1-5 years (D) 10-20 years (E) More than 25 years

(E) varicella

A 6-year-old unimmunized child has fever of 104°F (40°C) and cropped vesicles on the trunk with scattered scabbed lesions. Which of the following infections is the likely diagnosis? (A) measles (B) smallpox (C) HHV-6 (D) herpes simplex virus (E) varicella

(A) Serum bilirubin levels may be elevated. Stones are found in the gallbladder in over 90% of patients with cholecystitis. Bacteria are cultured in bile in about half the patients undergoing surgery; however, many patients have previously received antibiotics. The gallbladder is usually distended in patients with acute cholecystitis but contracted in chronic cholecystitis.

A 65-year-old woman is admitted with RUQ pain radiating to the right shoulder, accompanied by nausea and vomiting. Exam reveals tenderness in the RUQ and a positive Murphy's sign. A diagnosis of acute cholecystitis is made. What is the most likely finding? (A) Serum bilirubin levels may be elevated. (B) Cholelithiasis is present in 40-60%. (C) Bacteria are rarely found at operation. (D) An elevated amylase level excludes this diagnosis. (E) A contracted gallbladder is noted on ultrasound.

A) Bowel rest, nasogastric suction, IV fluids, and broad spectrum antibiotics

A patients CT scan reveals diverticulitis confined to the sigmoid colon. There is no associated abscess. What is the best course of treatment? A) Bowel rest, nasogastric suction, IV fluids, and broad spectrum antibiotics B) Urgent surgical resection C) Steroids D) Diverting colostomy E) Ileostomy

(C) neonatal acne

An 8 day old infant developed inflammatory papules and pustules on the forehead, nose, and malar areas of the face. The child is other- wise well, and the remainder of the physical examination is normal. Which of the follow- ing is the most likely diagnosis? (A) congenital syphilis (B) impetigo (C) neonatal acne (D) staphylococcal pustulosis (E) tuberous sclerosis

(B) watery diarrhea

An 8yo was hiking with his Boy Scout troop a few days ago. He now presents to your office with 2 days of abdominal pain. On further history, you realize he drank from the stream on his hike. You suspect Giardia intestinalis infec- tion. What additional symptom would you see most commonly in a child with giardiasis? (A) fever (B) watery diarrhea (C) bloody diarrhea (D) failure to thrive (E) vomiting

(B) 6-9 months

An infant is experiencing stranger anxiety. What is the typical age when this appears? (A) 4-6 months (B) 6-9 months (C) 15-18 months (D) 18-24 months (E) 24-36 months

B. metoprolol Of these Beta-blockers (which are usually AVOIDED) in reactive airway disease - metoprolol is the most "cardioselective", so theoretically could be used....although, on an exam, I would avoid beta-blockers in general.

Question 1 CORRECT A patient presents complaining of severe pain and "burning" in an extremity. You note that the extremity is pale and cool to the touch. You cannot appreciate a palpable pulsation. Which of the following diagnostic modalities will identify the source of this patient's problem in approximately 95% of cases? A chest x-ray B echocardiogram aortic angiogram D abdominal flat plate E aortic ultrasound Question 1 Explanation: Angiogram is the "gold standard" for occlusion of an arterial vessel. Question 2 CORRECT A 31 year-old pharmacist complaining of rectal pain. He describes the pain as "a severe tightness that awakens him from sleep." His bowel activity is normal. He denies rectal bleeding and seepage. He adds that sleep interruption is problematic, because with the number of hours he works, every minute of sleep is important. What is the most likely diagnosis? A anal abseess B perianal fistula proctalgia fugax D ulcerative colitis E internal hemorrhoids. Question 2 Explanation: proctalgia (rectal pain) fugax (comes and goes) is the best description. Abscess would be constant, fistula would drain, UC would cause bloody mucousy diarrhea, hemorrhoids would cause no pain, but bleeding. Question 3 CORRECT A 38 year-old chronic smoker presents with shortness of breath and wheezing. He has had several similar episodes in the past. He states that each previous episode began after developing a "cold that moved into his chest." Usually, after treatment with albuterol (VENTOLIN) and several days, the wheezing stops. He adds that he has a chronic cough, productive of mucous, most mornings during the past several years. Which of the following best describes this patient's condition? A chronic emphysema B chronic bronchitis chronic bronchitis with hypersensitive airways (asthmatic bronchitis) D cor pulmonale E bronchiectasis Question 3 Explanation: This is the best descriptor. Question 4 CORRECT A patient is being treated for Tuberculosis. She is experiencing central scotomata, a loss of green-red color perception and decreased visual acuity. Which agent is most likely responsible? A rifampin B isoniazid C streptomycin ethambutol E para-aminosalicylic acid Question 4 Explanation: Ethambutol is the TB drug that causes "E"ye symptoms. I remember it because it begins with an E. Question 5 CORRECT Secondary to a traumatic event, a child complains of pain in the index finger. An x-ray of the digit demonstrates a fracture line through the metaphysis of the proximal aspect of the middle phalanx, ending at the epiphyseal plate. What type of fracture does this child have? A Salter Harris Type I Salter Harris Type II C Salter Harris Type III D Salter Harris Type IV E Salter Harris Type V Question 5 Explanation: Salter I = slight increase in Space between epiphyseal plate and metaphysis Salter II = fx Above the plate (in the metaphysis) Salter III = fx Lower (in the epiphyseal plate) Salter IV = fit Through (both the metaphysic and epiphysis) Salter V = Really bad (comminuted fx compressing the epiphysis) This spells SALTR and may help you remember. 7365546_orig About Jorge Muniz PA-C (Creator of Medcomic) Question 6 CORRECT A 24 year-old male presents complaining of chest pain. He states that it is worse with swallowing and taking a deep breath. It is improved by sitting up and leaning forward. He denies trauma, a cough and shortness of breath. Which of the following tests would be most compatible with your suspected diagnosis? A a hiatal hernia visualized on chest x-ray B a normal erythrocyte sedimentation rate C calcified "popcorn" lesions in the lung fields bilaterally diffuse ST segment elevation on his electrocardiograph E a widened A-a gradient on his arterial blood gas Question 6 Explanation: This is pericarditis (by clinical presentation) which causes diffuse ST segment elevation on ECG (there can be notching of the R wave as well) Question 7 CORRECT A 2 month-old febrile male is brought to your facility to be evaluated for loss of appetite, irritability, and an acute petechial rash. Rectal temperature is 102.8F. Which of the following diagnostic studies is the most important in this child's evaluation? A white blood cell count and differential B urinalysis CSF analysis D serum glucose E chest X-ray (CXR) Question 7 Explanation: Any infant (neonate) with fever and rash should have a lumbar puncture (LP). While I would certainly do a CBC, even if it was normal, I would want the LP. Question 8 CORRECT Your 27 year-old sister is visiting and requests you to provide refills of dexamethasone and homatropine ophthalmic drops for her. What condition is most likely being treated? A conjunctivitis B glaucoma iritis D Herpes keratitis E blepharitis Question 8 Explanation: Iritis is treated with steroid drops (dexamethasone) and miotic drops *homatropine, like atropine (to constrict and fix the pupil to help the pain and open the angle until the iritis is resolved) Question 9 PARTIAL-CREDIT A 58 year-old male presents complaining of weakness of his grip. Your examination reveals that the problem is bilateral. During the next few office visits, you note the development of hyperactivity of his DTRs, extensor plantar reflexes and dysarthria. The patient's sensory system remains normal and he denies any urinary symptomatology. Which of the following is the most likely diagnosis? A multiple sclerosis B Alzheimer's disease Huntington's choreaHint: Huntington's causes a movement disorder with writhing choreiform movements of the body amyotrophic lateral sclerosis E myasthenia gravisHint: Myasthenia causes fatigue of the ocular muscles typically worsening at the end of the day. Question 9 Explanation: ALS (Lou Gehrig's disease) is a progressive bilateral muscle disease which causes fasciculations (lower motor neuron), and hyper-reflexia, plantar reflexes (upper motor neuron) and dysarthria. Sensation is normal as is bladder function. MS may have dysarthria, but reflexes are normal, sensation is impaired and bladder function is frequently affected. Alzheimer's has normal neuro exam with cognitive disability. Huntington's causes a movement disorder with writhing choreiform movements of the body. Myasthenia causes fatigue of the ocular muscles typically worsening at the end of the day. Question 10 CORRECT Which of the following is NOT a characteristic feature of multiple myeloma? A elevated serum calcium B osteoporosis C "punched out" osseous lesions D plasma cell infiltration of bone marrow hypogammaglobulinemia Question 10 Explanation: MM is a HYPERgammaglobulinemia - all of the other findings occur in MM. Question 11 PARTIAL-CREDIT Which of the following is NOT a risk factor for the development of osteoporosis? low testosterone levels in men low levels of physical activity inadequate dietary protein cigarette smoking E chronic corticosteroid use Question 11 Explanation: Low dietary Calcium, not protein, is a risk factor for osteoporosis. All of the others are risk factors. Question 12 A 12 year-old male presents complaining of no appetite for 24 hours and pain near his navel. During the night, the pain moved to the right lower abdomen. He is now nauseated and vomiting and has a low-grade fever. In the operating room, a normal appendix is discovered. What is the most likely diagnosis? A mesenteric ischemia B diverticulitis C mesenteric adenitis D cholecystitis E proctitis Question 13 A patient that must be on a beta-blocking agent has reactive airway disease and commonly experiences central nervous system side effects from medications. Which of the following beta-blockers would most likely be tolerated by this patient? A. atenolol B. metoprolol C. nadfilol D. propranolol E. pindolol

E) builds a tower of 8-10 cubes

The mother of a 2-year-old girl wonders if her daughter's motor skills are age appropriate. Which of the following is a motor skill that most 2-year-olds have attained? (A) stands on one foot for 10 seconds (B) climbs stairs using alternating feet (C) pedals a tricycle (D) copies a circle (E) builds a tower of 8-10 cubes

(C) at 2 months At birth, the retina is well developed but the lens is rather immobile. Fixation and tracking through the visual field are well developed by 2 months of age.

Vision milestones are being discussed with a group of student nurses. One student asks when babies can begin to visually follow people as they walk across the room.Fixation and tracking through the visual field are well developed by which of the following ages? (A) at 7 months gestation (B) at birth (C) at 2 months (D) at 6 months (E) at 1 year

D. IgA

Which of the following is passed to a baby during breastfeeding? A. IgE B. IgM C. IgG D. IgA

A.Paroxysmal supraventricular tachycardia

Which of the following is the most common ECG abnormality associated with mitral valve prolapse? A.Paroxysmal supraventricular tachycardia B. QT prolongation C. Rapid atrial fibrillation D. ST-segment depression in leads II, III, and aVF E. Ventricular tachycardia

C. 150 WBC, 80% lymphocytes

A 12-year-old presents to the emergency department in August with fever 102.2°F (39°C) and intense headache. A lumbar puncture (LP) is performed, and your suspicion of aseptic meningitis is confirmed. Which of the following CSF findings is most likely 48 hours into the course of enteroviral meningitis? A. 5,000 WBC, 90% polymorphonuclear leukocytes B. 100 WBC, 90% eosinophils C. 150 WBC, 80% lymphocytes D. 50 WBC, 90% polymorphonuclear leukocytes E. 50 WBC, 70% monocytes

(C) Application of direct pressure

A 12-yo boy is brought to the office with recurrent episodes of epistaxis during the past 2 months. No history of bruising or bleeding. While he is in the room, an episode of epistaxis occurs. On PE, anterior bleeding is visible within the Kiesselbach plexus. Which of the following is the most appropriate initial step in management? (A) Administration of clotting factors (B) Anterior nasal packing (C) Application of direct pressure (D) Cautery of the bleeding area (E) Referral of the patient to a pediatric otolaryngologist

(D) amoxicillin Amoxicillin is the first-line therapy for otitis media and acute sinusitis in children without penicillin allergy

A 14-year-old presents to your office with 2 week history of cough and purulent rhinorrhea. He now has 2-day history of fever and frontal headache. On examination he is tender to palpation over his maxillary sinuses and his mother tells you that he appears swollen over that area. You make the clinical diagnosis of acute sinusitis. He has no allergies to medication. What is the first-line therapy for this child? (A) ceftriaxone (B) cephalexin (C) clindamycin (D) amoxicillin (E) azithromycin

(C) parvovirus B19 erythema infectious (fifths disease)

A 15-month-old child presents to your office with a high fever, and an intense, red rash on the cheeks with circumoral pallor. What is the most likely etiology of this febrile exanthem? (A) enterovirus 71 (B) adenovirus (C) parvovirus B19 (D) rubeola virus (E) coxsackievirus A16

A. Collecting water from a stream, without boiling or chemical treatment Giardiasis is an infection of Giardia lamblia, a protozoan parasite that can cause epidemic or sporadic diarrhea. The major site of infection is the small intestine. While the exact etiology is uncertain, diarrhea may be a result of both intestinal malabsorption and hypersecretion. The small intestine is the site of the major structural and functional abnormalities associated with giardiasis.

A 15-year-old male with a recent history of camping presents after 5 days of flatulence and greasy, foul-smelling diarrhea. The patient also reports nausea, weight loss, and abdominal cramps which precedes the sudden diarrhea. He denies tenesmus, urgency, or bloody diarrhea. What is the patient most likely to report about his camping activities? A. Collecting water from a stream, without boiling or chemical treatment B. Recent antibiotic prescription C. Symptoms have been going on for months D. The patient camped as a side-excursion from a cruise ship E. The patient camped in Mexico

(B) Water-soluble contrast enema

A 16-month-old boy is brought to the emergency department because of 2- to 3-minute episodes of acute abdominal pain over the past 6 hours. During the episodes, he turns pale, screams, and draws up his knees. He has also passed one bloody stool. A mass is palpated in the right upper quadrant of the abdomen. Which of the following is the most appropriate next step in management? (A) Nothing by mouth; administration of fluids and antibiotics, intravenously (B) Water-soluble contrast enema (C) CT scan of the abdomen (D) Colonoscopy (E) Laparotomy

(E) Substance use (substance abuse) Substance abuse in teens is manifest by a change in personality traits, decreased school performance, neglected hygiene, and a change in peer group preferences.

A 16-yo boy is brought to the office because of decreasing performance in school over the past 6 months. During this time, the pt has become more irritable and irresponsible, has changed his group of friends, and has had decreasing personal hygiene. He was previously a high-achieving student, but his grades have slipped to the extent that he is failing courses. Which of the following disorders is the most likely cause of this patient's symptoms? (A) Bipolar (B) Conduct (C) Major depressive (D) Persistent depressive (dysthymic) (E) Substance use (substance abuse)

E. increased atypical lymphocytes on white blood cell differential With a negative rapid strep screen, the most likely explanation for this presentation is acute infectious mononucleosis. The fever, fatigue, tonsillar hypertrophy, and splenomegaly are all classic symptoms and signs. Laboratory evaluation often includes an elevated total white blood cell count with increased atypical lymphocytes on differential.

A 16-yo girl complains of a very sore throat, swollen lymph nodes, fever, & general malaise. Her exam reveals a temp of 102.2°F, enlarged exudative tonsils, tender cervical lymphadenopathy, and borderline enlarged spleen. Rapid strep is negative. Which of the following laboratory findings best supports the most likely diagnosis? A. decreased white blood cell count B. increased monocytes on white cell differential C. thrombocytosis D. decreased levels of antibody to Epstein-Barr viral capsid antigen E. increased atypical lymphocytes on white blood cell differential

C. anion gap metabolic acidosis with respiratory alkalosis Salicylates affect most organ systems, leading to various metabolic abnormalities. Because salicylates are a gastric irritant, symptoms of vomiting and diarrhea occur soon after the overdose, which may contribute to the development of dehydration. Salicylates stimulate the respiratory center leading to hyperventilation and hyperpnea resulting in respiratory alkalosis and compensatory alkaluria. *A characteristic feature of salicylate intoxication is the coexistence of a respiratory alkalosis with a widened anion gap metabolic acidosis.*

A 16-yo reportedly ingesting "a bottle of aspirin." Vitals are temp 37.8°C oral; pulse 94/min; resp 30/min; BP 100/68 mm Hg. What would you expect her blood gases to show? A. anion gap metabolic acidosis with respiratory acidosis B. nonanion gap metabolic acidosis with respiratory alkalosis C. anion gap metabolic acidosis with respiratory alkalosis D. nonanion gap metabolic acidosis with respiratory acidosis

(C) Exploratory laparotomy

A 19-year-old man presents to the emergency department with a gunshot wound through the umbilicus. The systolic blood pressure is 70 mm Hg on palpation, and his abdomen is tightly distended. Large-bore intravenous lines are placed, and Ringer's lactate is infused. What should be the next step? (A) Peritoneal lavage (B) CT scan of the abdomen (C) Exploratory laparotomy (D) Transfusion of the patient until the systolic blood pressure reaches 90 mm Hg (E) PASG

C. chlamydial Chlamydial infections are the most common cause of conjunctivitis in newborns in developed countries.

A 2-wk-old male is being seen in the clinic for a profuse mucoid discharge from both eyes, with some tearing. Both eyes are hyperemic and the eyelids are red and swollen. Which of the following is the most likely cause of this patient's ophthalmia neonatorum (conjunctivitis in the newborn)? A. allergic B. gonococcal C. chlamydial D. viral

(C) B12 Because vitamin B12, or cyanocobalamin, is mainly found in animal products, individuals who follow a strict vegan diet commonly have deficiency of this nutrient. In breast- feeding mothers who follow a vegan diet, the breast milk supply is likely to reflect this deficiency. Because the patient described is breast-fed and his mother follows a vegan diet, he is at greatest risk for deficiency of vitamin B12.

A 2-wk-old male newborn is brought to the office by his mother for routine well-child exam. The patient is breast-fed, and his mother follows a vegan diet. The mother says she plans to breast-feed until the patient reaches 6 months, and then she will introduce him to solid foods that conform to her vegan diet. On the basis of this information, the patient is at greatest risk for deficiency of which of the following vitamins? (A) B1 (B) B6 (C) B12 (D) C

(E) Spontaneous regression has occurred in some children.

A 2-year-old child is referred to you for evaluation of child abuse. On physical examination, you find a pale child with diffuse petechiae and bilateral proptosis with periorbital ecchymoses. Which of the following statements is true about this condition? (A) Age at presentation correlates directly with survival. (B) A full skeletal survey should be obtained next. (C) Hematuria is a common finding. (D) It usually presents between 4 and 8 years of age. (E) Spontaneous regression has occurred in some children.

D. Parainfluenza virus This child has the classic presentation for croup. Sixty-five percent of croup is caused by a parinfluenza virus infection.

A 2-year-old girl has developed a "barking" cough and a low-grade fever. She has some runny nose and her voice is somewhat raspy and hoarse. When approached by the PA, she becomes somewhat upset and exhibits mild inspiratory stridor. She appears otherwise well and has no drooling or dyspnea. What is the most likely etiology of this child's illness? A. Adenovirus B. Coxsackievirus C. Cytomegalovirus D. Parainfluenza virus E. Respiratory syncytial virus

(A) hepatic injury

A 2-year-old is found with an opened empty bottle of acetaminophen tablets and has pill fragments in his mouth. The major cause of morbidity and mor- tality in acute poisoning with acetaminophen is which of the following? (A) hepatic injury (B) gastric bleeding (C) metabolicacidosis (D) methemoglobinemia (E) hypoglycemia

(E) Wilms tumor clinical presentation of hematuria, abdominal pain of two months' duration, and a palpable mass in the right upper quadrant of the abdomen is characteristic of this tumor.

A 2-yo girl is brought in after blood was noticed in her urine. Parents say the pt has had intermittent abdominal pain during the past 2 months but has been otherwise well. On PE, the abdomen is slightly distended and a mass is palpated in the RUQ. UA is positive for blood and protein. Which of the following is the most likely diagnosis? (A) Cystic nephroma (B) Cystitis (C) Mesoblastic nephroma (D) Neuroblastoma (E) Wilms tumor

A. atrial fibrillation

A 21-year-old male is brought to the emergency department by his fraternity brothers because he is complaining about his heart "trying to jump out of my chest." He had consumed a large amount of alcohol over the course of the day. He is obviously intoxicated and quite anxious. ECG shows erratic atrial activity with irregular ventricular response. What is the most likely diagnosis? A. atrial fibrillation B. atrial flutter C. paroxysmal supraventricular tachycardia D. ventricular premature beats

C. Delayed primary closure or healing by secondary intention is appropriate

A 21-yoman presents with lacerations over the second and third metacarpophalangeal joints of his left hand after being involved in a fistfight the previous evening. Which of the following statements regarding his treatment is correct? A. Absorbable sutures should be used because he might not get followup care B. Antibiotics are not indicated because the likelihood of infection is low C. Delayed primary closure or healing by secondary intention is appropriate D. First-generation cephalosporins should be used as first-line treatment

B) Vitamin B12

A 23 year old patient presents to your office for preconception counseling. She mentions that she is a vegetarian. Which vitamin is most commonly deficient in vegetarian diets? A) Folate B) Vitamin B12 C) Vitamin C D) Vitamin A

D. 7

A 23-yo man is brought in by ambulance after falling off a roof. He is moaning and has sonorous respirations; he will not open his eyes to pain but withdraws to pain when tested. What is his Glasgow Coma Scale score? A. 4 B. 5 C. 6 D. 7 E. 8

D. diffuse ST segment elevation on his electrocardiograph This is pericarditis (by clinical presentation) which causes diffuse ST segment elevation on ECG (there can be notching of the R wave as well)

A 24 year-old male presents complaining of chest pain. He states that it is worse with swallowing and taking a deep breath. It is improved by sitting up and leaning forward. He denies trauma, a cough and shortness of breath. Which of the following tests would be most compatible with your suspected diagnosis? A. a hiatal hernia visualized on chest x-ray B. a normal erythrocyte sedimentation rate C. calcified "popcorn" lesions in the lung fields bilaterally D. diffuse ST segment elevation on his electrocardiograph E. a widened A-a gradient on his arterial blood gas

(A) Percutaneous drainage of the cyst

A 24-year-old college student recovers from a bout of severe pancreatitis. He has mild epigastric discomfort, sensation of bloating, and loss of appetite. Examination reveals an epigastric fullness that on ultrsound is confirmed to be a pseudocyst. The swelling increases in size over a 3-week period of observation. What should be the next step in management? (A) Percutaneous drainage of the cyst (B) Laparotomy and internal drainage of the cyst (C) Excision of pseudocyst (D) Total pancreatectomy (E) Administration of pancreatic enzymes

(B) A complete history and physical examination

A 24-year-old woman is scheduled for an elective cholecystectomy. The best method of identifying a potential bleeder is which of the following? (A) Platelet count (B) A complete history and physical examination (C) Bleeding time (D) Lee-White clotting time (E) Prothrombin time (PT)

A. Abdominal CT pt is stable at this time and can tolerate a CT

A 24-yo man presents after being struck on the left side by a pickup truck while crossing a street. Initial vital signs are BP 130/78, pulse 110, and RR 18. He is alert & oriented and had no loss of consciousness at the scene. Paramedics have established two large-bore peripheral lines and are infusing normal saline; he has received less than 500 mL on arrival. Abdominal examination reveals tenderness in the LUQ and suprapubic region. Plain radiographs reveal a pelvic fracture. What would be the best diagnostic test in this scenario to definitively diagnose the etiology of the patient's abdominal pain? A. Abdominal CT B. Diagnostic peritoneal lavage C. Exploratory laparotomy D. FAST exam E. Retrograde urethrography

(B) The disease is confluent, there are no skip areas in the colon and the rectum is involved.

A 25-year-old male develops diarrhea and colicky abdominal pain. Ulcertive colitus is diagnosed on colonoscopy. Which of the following findings is consistent with the diagnosis? (A) The rectum is not involved. (B) The disease is confluent, there are no skip areas in the colon and the rectum is involved. (C) The full thickness of the bowel wall is involved. (D) Microscopic examination of the mucosa reveals normal cells without evidence of dysplasia. (E) The incidence of colorectal cancer is equal to that of the general population.

(A) Traumatic rupture of the diaphragm

A 25-year-old man arrives in the emergency department in respiratory distress following a motor vehicle collision. A chest x-ray shows abdominal viscera in the left thorax. What is the most likely diagnosis? (A) Traumatic rupture of the diaphragm (B) Sliding esophageal hernia (C) Short esophagus with intrathoracic stomach (D) Rupture of the esophagus (E) Bochdalek hernia

(A) Crohn's colitis

A 25-year-old man has recurrent, indolent fistula in ano. He also complains of weight loss, recurrent attacks of diarrhea with blood mixed in the stool, and tenesmus. Proctoscopy revealed a healthy, normal-appearing rectum. What is the most likely diagnosis? (A) Crohn's colitis (B) Ulcerative colitis (C) Amoebic colitis (D) Ischemic colitis (E) Colitis associated with acquired immunodeficiency syndrome (AIDS)

D. perform an immediate needle decompression

A 25-yo sustained significant blunt force trauma to the chest in a MVC. He arrived at the emergency department on supplemental oxygen. An initial evaluation, including CXR, revealed three rib fractures but no other injuries. Suddenly, the patient reports severe shortness of breath. Vital signs at this time are BP 106/66, pulse 20, SaO2 85%. What is the next best step in management of this patient? A. increase supplemental oxygen B. intubate immediately C. order a STAT CT scan of chest D. perform an immediate needle decompression

(D) H. pylori infection

A 26-year-old man is diagnosed with adeno- carcinoma of the stomach. He wants to know what could have caused him to develop this condition. He does an internet search. Which of the following is a risk factor for developing gastric cancer? (A) Exposure to ionizing radiation (B) Blood group B (C) A diet high in fiber (D) H. pylori infection (E) North American descent

(B) Wound exploration Wound exploration that convincingly doc- uments failure of penetration through the pos- terior rectus fascia will most likely exclude abdominal injury from a tangential gunshot wound.

A 26-year-old man presents with a tangential small-caliber gunshot wound of the anterior abdominal wall. (A) Peritoneal lavage (B) Wound exploration (C) Sonogram (D) Paracentesis (E) CT with intravenous and oral contrast (F) IVP (G) Exploratory laparotomy (I) Angiogram

B. Hypothalamic A young healthy woman with a low BMI, no other signs of virilization, and a history of normal menarche is most likely to have induced amenorrhea, due to suppression of the hypothalamic axis from low weight and fat index.

A 26-yo athlete presents complaining of scant menses x 4 months. She is a G0P0, menarche at age 13, and her menses have been mostly regular. She is a non-smoker and non-drinker, and has been trying to achieve pregnancy x 8 months. PE reveals a thin, white female in no distress. Vitals are normal, BMI is 17.5, her pelvic exam is normal, and STD cultures are negative. A pregnancy test is negative. What is the most likely cause for her amenorrhea? A.Idiopathic B. Hypothalamic C. Hypothyroid D. Polycystic ovarian syndrome E. Androgenic

C. pregnancy will increase her risk of abuse

A 26-yo female presents to the ED with multiple bruises over her face, arms, and trunk. She tells you that she slipped and tumbled down a flight of stairs. The patient is extremely anxious and her contusions are not consistent with her reported fall.When you question her further, she tells you that her husband sometimes drinks too much and she needs to get home before he wakes up. Checking the patient's medical record, you find that this is her third visit in the last year for similar injuries.Which of the following is most likely true about this patient and her situation? A. family therapy is usually not very helpful B. leaving her home will reduce her risk of injury and death C. pregnancy will increase her risk of abuse D. it is unlikely that she grew up in a violent home

E. Torsion of the left ovary

A 29-yo woman comes to the ED with abrupt onset of LLQ pain approx 1.5 hours ago after she and her roommate moved heavy pieces of furniture into their apartment. The pain is now excruciating and she rates it "100 /10." She vomits on admission. She is not currently sexually active and has no other significant medical history. On exam she is found to have fullness and extreme tenderness in the left adnexa. what is the most likely diagnosis? A. Diverticulitis B. Pelvic inflammatory disease C. Renal calculus D. Ruptured ovarian cyst E. Torsion of the left ovary

C. Impetigo; PO Cephalexin Honey crusted lesions with no lesions on oral mucosa or elsewhere that are pruritic, and "going around" in day care leads to the most likely diagnosis of Impetigo Most common etiology is S. Aures Given that there are multiple lesions it is recommended to treat with Dicloxacillin or Cephalexin PO; If less severe might consider Topical Mupirocin or Retapamulin

A 3 yo boy presents with a rash around his mouth that his dad says other kids at daycare have gotten as well. No lesions on the oral mucosa or on any other locations on body. Father denies any fever, N/V/D, and hasn't noticed any increased fatigue. The child's father keeps stopping his son from itching the lesions. What is the most likely diagnosis & treatment A. Herpes Simplex Virus 1; Topical Acyclovir B. Coxsackievirus; PO Amoxicillin C. Impetigo; PO Cephalexin D. Impetigo; PO Amoxicillin E. Herpes Simplex Virus 1; PO Acyclovir

A. ceftriaxone Gonococcal ophthalmia neonatorum presents as a unilateral or bilateral serosanguineous discharge and then within 24 hours the discharge becomes mucopurulent, followed by conjunctival injection and edema of the eyelids.

A 3-day-old infant has bilateral copious, yellow-green eye discharge and conjunctival inflammation. A Gram stain of this discharge reveals gram-negative intracellular diplococci. Which of the following antibiotics is the drug of choice for this infection? A. ceftriaxone B. cephalexin C. erythromycin D. gentamicin

(B) rotavirus

A 3-year-old child is admitted to the hospital in February with temperature of 101.3°C (38.5°C), vomiting, and diarrhea. The mother describes the stools as watery, green, and very foul smelling. The most common organism causing this presentation is (A) adenovirus (B) rotavirus (C) Salmonella enteriditis (D) Shigella sonnei (E) enterovirus

(E) an organophosphate

A 3-year-old child who is unresponsive presents with weakness, excessive salivation, bradycardia, and constricted pupils. The parents are so distraught, it is difficult to get information. However, an astute emergency room physician realizes the most likely drug or toxin to cause these signs is which of the following? (A) diphenhydramine (B) phenobarbital (C) ethylalcohol (D) a hydrocarbon (E) an organophosphate

(B) deferoxamine (desferal)

A 3-year-old female was found with her mother's prenatal vitamins. The bottle was empty and was estimated to contain 20-25 tablets. At 4 hours postingestion, her serum iron level is 550 mg/dL. Which of the following should be administered? (A) dimercaprol(BAL) (B) deferoxamine (desferal) (C) edetate calcium disodium (EDTA) (D) ipecac (E) activatedcharcoal

(E) Sweat chloride test The clinical scenario described includes characteristic symptoms of *cystic fibrosis*, including growth delay, multiple upper and lower respiratory tract infections, and malabsorption Sweat Chloride test is Gold Standard for diagnosis of CF

A 3-yo boy is seen for a new Pt well-child exam. Height is in the 3rd percentile, and weight is in the 1st percentile. Parents say the patient has been treated multiple times since infancy for sinus infections & pneumonia. They also note his stools are generally loose, greasy, & mucousy. During PE, the pt coughs frequently. No other abnormalities are noted. Which of the following studies is most effective to determine the diagnosis? (A) Bronchoscopy (B) CT scan of the sinuses (C) Culture of aspirate from the trachea (D) Measurement of serum immunoglobulin levels (E) Sweat chloride test

(B) obtain a chest x-ray Up to 28,000 children (usually < 5 years of age) ingest hydrocarbons each year. Most chil- dren will remain asymptomatic. Those who develop respiratory symptoms, even if transient, should be evaluated by a physician. A chest radi- ograph should be part of the initial evaluation of all those children who developed respiratory symptoms, such as wheezing, coughing, gagging, or dyspnea, even if symptoms were transient.

A 3-yo is brought to the ED 3 hours after he was found playing with an open kerosene bottle. The parents state the child initially had some gagging and coughing and within an hour developed labored breath- ing. At the time of your evaluation though, the child's examination is completely normal. What is the most appropriate action at this time? (A) discharge to home, advise parents to return if they note any problems (B) obtain a chest x-ray (C) observe child in emergency room for 1 hour (D) induce emesis with syrup of ipecac (E) admit to the hospital for 24 hours of observation

(C) Neurogenic shock

A 30-year-old man is brought to the emergency department following a high-speed car accident. He was the driver, and the windshield of the car was broken. On examination, he is alert, awake, oriented, and in no respiratory distress. He is unable to move any of his four extremities; however, his extremities are warm and pink. His vital signs on admission are HR 54 bpm and BP 70/40 mm Hg. What is the diagnosis? (A) Hemorrhagic shock (B) Cardiogenic shock (C) Neurogenic shock (D) Septic shock (E) Irreversible shock

(C) They can become damaged, even though urine output exceeds 1500 mL/d.

A 35-year-old man is admitted with systolic blood pressure (BP) of 60 mm Hg and a heart rate (HR) of 150 bpm following a gunshot wound to the liver (Fig. 1-1). What is the effect on the kidneys? (A) They tolerate satisfactorily ischemia of 3-4 hours duration. (B) They undergo further ischemia if hypothermia is present. (C) They can become damaged, even though urine output exceeds 1500 mL/d. (D) They are affected and cause an increased creatinine clearance. (E) They are prevented from further damage by a vasopressor.

D. progesterone 10 mg/day for 10 days

A 39 yo female presents to the ED with vaginal bleeding x 3wks. She normally has regular periods, but she hasn't had a period in 2 mo prior to this. She denies abd pain and states she is soaking 4 pads/day. Her VS are normal, pregnancy is neg, her hematocrit is 40. Which of the following is the most appropriate next step in management? A. acetaminophen 600 mg every 6 hrs B. f/u with gynecologist in 2 wks C. IV estrogen D. progesterone 10 mg/day for 10 days E. STAT pelvic US

B. Montelukast Montelukast (B) is indicated for use in children 1 year of age or older zafirlukast (D) is indicated for children > 5 years of age, zileuton (E) is indicated for children > 12 years of age. Beclomethasone (A) is an inhaled corticosteroid and salmeterol (C) is an inhaled long-acting beta-2 agonist.

A 4 year-old girl presents to the clinic due to severe allergic rhinitis and recurrent asthma exacerbations. Which of the following leukotriene pathway inhibitors is indicated for use in this child? A. Beclomethasone B. Montelukast C. Salmeterol D. Zafirlukast E. Zileuton

(D) Kerion because the clinical presentation of inflammatory pustules with a surrounding area that is boggy and tender is most consistent with this condition.

A 4 yo boy is brought to the clinic with an area of sores on his scalp. Weight is 19.1 kg (42 lb). Vital are WML. PE shows a grouping of pustules in the right parietal region. The area surrounding this grouping is boggy and very tender, and annular hair loss is noted in that region. The cervical lymph nodes are diffusely tender and enlarged. No other abnormalities are noted. Which is the most likely diagnosis? (A) Alopecia areata (B) Atopic dermatitis (C) Herpes simplex virus (D) Kerion (E) Mastocytosis

E. Ventricular septal defect

A 4-month-old male presents for a well child check. He is healthy, and the mother feels that the child is eating and growing well. On exam, there is no cyanosis. The peripheral pulses are normal & equal. There is a medium-pitched harsh pansystolic murmur heard best at the L-sternal border at the 4th ICS. There is no heave or thrill. The murmur radiates over the entire precordium and the S2 is physiologically split. What is the most likely diagnosis? A. Atrial septal defect B. Coarctation of the aorta C. Patent ductus arteriosus D. Tetralogy of fallot E. Ventricular septal defect

A. Kawasaki syndrome

A 4-year-old boy presents with 5 days of fever, conjunctivitis, strawberry tongue, red lips, and injected throat. He has large, swollen, slightly tender lymph nodes in his neck and a peeling rash in the palms and soles. The most likely cause is A. Kawasaki syndrome B. respiratory syncytial virus C. coxsackievirus D. fifth disease

(B) Henoch-Schönlein purpura

A 4-year-old male presents with a 1-day history of abdominal pain and vomiting. He is afebrile and has no diarrhea. He complains of knee pain bilaterally, and there is some tenderness of the knee joints but no effusions. Within 24 hours he develops a rash on his legs and buttocks which is petechial and pupuric, and his platelet count is normal. What is the most likely diagnosis? (A) hemolytic uremic syndrome (B) Henoch-Schönlein purpura (C) acute glomerulonephritis (D) Kawasaki disease (E) systemic lupus erythematosi

(C) Enzyme immunoassay of stool the clinical scenario described is characteristic of *giardiasis*, and the most appropriate next step is a stool study that would confirm the causative organism of Giardia lamblia.

A 4-yo boy is brought in by his parents with intermittent nonbloody diarrhea & constipation along with abdominal pain and bloating during the past 3 weeks. When the patient's symptoms first began, his stools were watery, but they are currently greasy and foul. No fever or vomiting. Other children at day care have had similar symptoms. Vital signs are WNL. PE shows vague tenderness to palpation of the abdomen without rebound or guarding. Which of the following studies is the most appropriate next step? (A) Air contrast enema (B) Colonoscopy (C) Enzyme immunoassay of stool (D) Esophagogastroduodenoscopy (E) Fecal leukocyte test

(B) Measure blood pressure in the lower extremities Hypertension is children should raise suspicion for Coarction of the Aorta

A 4-yo girl is seen for her well-child exam. The parents say the pt has been healthy, but they have noticed that she tires more quickly than her peers when playing. She is in the 10th percentile for height and the 40th percentile for weight. BP is 132/82 mmHg in the left arm and 128/80 mmHg in the right arm. Repeat measurements are in the same range. Which is the most appropriate next step? (A) Counsel the parents regarding diet and exercise (B) Measure blood pressure in the lower extremities (C) Order urinalysis (D) Recheck blood pressure in two weeks (E) Refer the patient to a nephrologist

(B) Methanol ingestion

A 40-year-old man is found to have severe metabolic acidosis with a high anion gap. What is the most likely cause? (A) Diarrhea (B) Methanol ingestion (C) Proximal renal tubular acidosis (D) Distal renal tubular acidosis (E) Ureterosigmoidostomy

A. Broad-spectrum antibiotics This patient has Boerhaave syndrome, an esophageal perforation following a sudden rise in intraesophageal pressure. The mechanism is sudden, forceful emesis in about 75% of cases.

A 40-year-old man presents with severe chest and neck pain. He is otherwise healthy but says that he "threw up really bad" 6 hours earlier at a tailgate party. The neck pain is made worse by swallowing and by flexing his neck. What is the appropriate next management step? A. Broad-spectrum antibiotics B.Endoscopy C. Laboratory testing, to include lipase D. Soft-tissue neck x-ray E. Treatment with H2 blockers

(D) Determination of serum albumin is necessary

A 45-year-old male with a known history of alcoholism is admitted with acute pancreatitis. His serum calcium is 7 mg/dL. Management is based upon which of the following? (A) One-fourth of calcium in serum is ionized (B) Alkalosis increases the ionized calcium component (C) Hypocalcemia may cause polyuria and polydypsia (D) Determination of serum albumin is necessary (E) Treatment should involve intravenous administration of calcium chloride

(E) Upper endoscopy and biopsy

A 45-year-old man complains of burning epigastric pain that wakes him up at night. The pain is relieved by eating or using over-the-counter antacids and H2 blockers. Diagnosis is best confirmed by which of the following? (A) Urea breath test (B) Serum gastrin levels (C) Barium meal examination (D) Upper endoscopy (E) Upper endoscopy and biopsy

(C) Arteriovenous (AV) fistula

A 45-year-old woman undergoes cardiac catheterization through a right femoral approach. Two months later, she complains of right lower extremity swelling and notes the appearance of multiple varicosities. On examination, a bruit is heard over the right groin. What is the most likely diagnosis? (A) Femoral artery thrombosis (B) Superficial venous insufficiency (C) Arteriovenous (AV) fistula (D) Pseudoaneurysm (E) Deep vein insufficiency

E. RUQ ultrasonography

A 46-yo woman presents with constant abdominal pain with associated nausea that started 8 hours earlier. She appears nontoxic and is lying still on the bed. BP is normal; pulse rate 95, RR is 16, and temp is 37.8°C (100°F). Physical examination is remarkable for RUQ and epigastric tenderness to palpation without rebound or guarding. Lab test results reveal elevated ALT, AST, and alkaline phosphatase, normal lipase and total bilirubin, and a negative urine hCG. The next most appropriate management step is: A. Acute abdominal series B. CT scan C. General surgery consultation D. Oral cholecystography E. RUQ ultrasonography

A. Aortic stenosis

A 49-yo man presents after he fainted while running on his treadmill at home. He has been having exertional dyspnea and angina for the past several months. Which of the following cardiac diseases is most likely to cause these symptoms? A. Aortic stenosis B. Atrial septal defect C. Mitral incompetence D. Pulmonary stenosis E. Tricuspid incompetence

(B) Reassure the mother that this is a common behavior and is not abnormal. Pediatric masturbation is a very common behavioral concern but is also entirely normal. If parents are worried about it being a social problem encourage the parents to tell their children to "only do it in their room" or "only do it in the house". Most younger children grow out of this once they start kindergarten.

A 4yo girl presents with her mom for her well child visit. Her PE is completely normal. Her mom says she is perfectly healthy but does have one concern. She has noticed that her daughter often puts her hands in her pants and touches her privates when she is going to sleep. Her mother is concerned that this is abnormal. Which of the following is the best intervention for this behavior? (A) Refer the child to a child behavioral therapist for immediate intervention (B) Reassure the mother that this is a common behavior and is not abnormal (C) Recommend that the mother start punishing her child for this behavior through negative punishment (D) Call child protective services and report suspected child abuse

(A) Coarctation of the aorta

A 5-day-old neonate is brought to the ED 20 min after sudden onset of irritability, diaphoresis, and profound dyspnea. No fever or other symptoms of systemic illness. On PE, auscultation of the chest shows a grade 2/6 systolic ejection murmur that is heard best at the LUSB and radiates to the left interscapular area. Palpation of the abdomen shows enlarged liver. Femoral pulses are absent bilaterally, and the lower extremities appear cyanotic compared with the upper extremities. No other abnormalities are noted. Which is the most likely diagnosis? (A) Coarctation of the aorta (B) Diaphragmatic hernia (C) Group B streptococcal sepsis (D) Patent ductus arteriosus (E) Tetralogy of Fallot

(D) phenylketonuria(PKU)

A 5-week-old male presents with poor feeding, poor growth, a peculiar odor, hypertonia, and hyperactive reflexes. He is afebrile. History reveals no problems with labor and delivery and early hospital discharge at 24 hours of age. He has not seen a physician since that time. Which of the following is the most likely etiology of this infant's condition? (A) sepsis (B) pyloric stenosis (C) overfeeding (D) phenylketonuria(PKU) (E) hypothyroidism

D. DTaP, IPV, MMR, varicella I really hope there aren't questions this specific about childhood vaccinations on the PANCE. But just in case...

A 5-year-old male child presents to the office for his kindergarten physical examination. Assuming that the patient's immunizations have been up to date, which of the following are the immunizations that the patient should receive at the end of today's visit? A. hepatitis B, inactivated poliovirus (IPV), diphtheria, tetanus, acellular pertussis (DTaP), measles, mumps, rubella (MMR), varicella B. IPV, DTaP, MMR, pneumococcal (PCV) C. IPV, DTaP, MMR, Haemophilus influenzae type B (Hib) D. DTaP, IPV, MMR, varicella

(D) give DTaP and tetanus immune globulin

A 5-year-old unimmunized child fell while playing in an old barn and sustained a laceration to his leg. After local wound care, what would be the most appropriate management regarding tetanus prophylaxis? (A) give Td only (B) give DTaP only (C) give Td and tetanus immune globulin (D) give DTaP and tetanus immune globulin (E) give tetanus immune globulin only`

(A) Wound dehiscence A large amount of seroanguinous drainage from the abdominal wound that occurs 5 to 7 days post-op is usually the result of dehiscence of the abdominal wound closure. A wound infection is heralded by erythema, swelling, and thick pus. Leaks from either enteric suture line would probably be bilious. Ascites is not commonly blood tinged.

A 51-year-old woman underwent a Billroth II subtotal gastrectomy for carcinoma of the stomach 6 days ago. She had been recovering well except for persistent ileus. On morning rounds, you notice a large amount of serosan- guinous drainage on her gown. The most likely diagnosis is: (A) Wound dehiscence (B) Wound infection (C) Leak at the gastrojejunostomy anastomosis (D) Leak from the duodenal stump (E) Ascites

E. Start cefotaxime 2 g IV q8h, and admit

A 53-yo man with cirrhosis presents with a 12-hr history of diffuse abdominal pain. Physical examination reveals a positive fluid wave, mild diffuse abdominal tenderness, and a temperature of 38°C (100.4°F). Paracentesis reveals 2,000 WBCs/microliter and 280 PMNs/microliter. Which of the following is the most appropriate management? A.Administer oral antibiotics, and discharge home B. Culture ascitic fluid, and treat if cultures are positive C. Order triple-contrast abdominal CT to look for causes of abdominal pain D. Perform RUQ ultrasonography to look for cholecystitis E. Start cefotaxime 2 g IV q8h, and admit

D. Surgical exploration

A 54-yo man presents to the ED with crampy abdominal pain, nausea, and vomiting. The patient has not passed gas or had a BM for at least 10 hours. On examination, the abdomen is distended and there are high-pitched bowel sounds with rushes. A plain radiograph of the abdomen reveals cecal distension to 12 cm. What is the most appropriate definitive management for this patient? A. IV fluids B. NG suction C. Observation D. Surgical exploration

(B) Analysis of arterial blood gas

A 55-year-old man sustains numerous injuries involving the abdomen and lower extremities. During the intra- and postoperative periods, he is resuscitated with 10 L of Ringer's lactate and 2 U of packed red blood cells (RBC). After initial improvement, he has severe dyspnea on the second postoperative day. The most useful initial diagnostic test is which of the following? (A) Electrocardiogram (B) Analysis of arterial blood gas (C) Insertion of a central venous line (D) Ventilation-perfusion scan (E) Computed tomography (CT) scan of abdomen

(C) Barium enema Patients who have symptoms suggestive of change in bowel habits will require a barium enema or colonoscopy. It is important not to overlook an underlying carcinoma, which could cause the patient to strain and induce a hernia. Carcinoma and/or polyps may be over- looked if this approach is ignored.

A 56-year-old man is scheduled to have a left indirect hernia repaired. He is asymptomatic. Before surgical treatment, he should have which of the following? (A) Rectal examination alone (B) Rectal examination and sigmoidoscopy (C) Barium enema (D) Colonoscopy (E) Intravenous pyelogram

(B) Pericardial tamponade

A 57-year-old hospitalized man undergoes right subclavian venous catheterization for hyperalimentation. He is currently being treated for a small bowel fistula. While the results of an x-ray of the chest to check the catheter position are pending, the patient suddenly becomes agitated. His pulse is 110/min, and blood pressure is 70/50 mm Hg. Examination shows jugular venous distention. The lungs are clear to auscultation. Breath sounds are equal bilaterally. The trachea is midline. An x-ray of the chest shows a catheter in the superior vena cava, transversing the right ventricle and crossing to the left of the midline. Which of the following is the most likely cause of this patient's hypotension? (A) Air embolism (B) Pericardial tamponade (C) Pulmonary artery perforation (D) Staphylococcal bacteremia (E) Tension pneumothorax

D. ALS

A 58 year-old male presents complaining of weakness of his grip. Your examination reveals that the problem is bilateral. During the next few office visits, you note the development of hyperactivity of his DTRs, extensor plantar reflexes and dysarthria. The patient's sensory system remains normal and he denies any urinary symptomatology. Which of the following is the most likely diagnosis? A. multiple sclerosis B. Alzheimer's disease C. Huntington's chorea D. ALS E. myasthenia gravis

(B) ceftriaxone and vancomycin Meningitis or septicemia potentially caused by S pneumoniae should be initially treated with a third-generation cephalosporin plus either vancomycin.

A 6-month-old infant presents from day care with fever, vomiting, and lethargy. On physical examination, you confirm the lethargy and note a full fontanel despite signs of dehydration. Which of the following would be the initial antibiotic(s) of choice? (A) cefuroxime and vancomycin (B) ceftriaxone and vancomycin (C) clindamycin (D) vancomycin (E) amoxicillin

B. 2 months Patients with sickle cell disease develop functional asplenia as early as 3 months of age and should begin treatment with prophylactic penicillin at 2 months of age to prevent infection by encapsulated organisms

A 6-week-old male with sickle cell disease presents to the pediatric office for his well-child visit. When should this child begin taking daily prophylactic penicillin? A. Now B. 2 months C. 6 months D. 12 months E. 5 years

(E) Transient synovitis the clinical presentation of pain in the knee, mild limp, limited internal rotation of the hip, and slightly elevated sedimentation rate with normal white blood cell count is most consistent with this condition.

A 6-yo boy has had pain in his right knee for the past two days. The pt walks with a mild limp. He has no history of trauma to the knee. He had cold symptoms one week ago but is otherwise healthy. VS are WNL. PE shows full ROM of the right knee with limited internal rotation of the right hip. ESR is slightly elevated and WBC count is WNL. X-rays show no abnormalities. Which is the most likely cause of the pain in this pt's knee? (A) Developmental dysplasia of the hip (B) Legg-Calvé-Perthes disease (C) Septic joint (D) Slipped capital femoral epiphysis (E) Transient synovitis

D. Legg-Calvé-Perthes disease Legg-Calvé-Perthes disease is also known as avascular necrosis of the proximal femur. It typically occurs in children between 4 and 8 years old and persistent hip pain is the main symptom.

A 6-yo female complains of chronic hip pain so severe that she can't walk to the bus. Exam shows severe tenderness at the hip with decreased active & passive ROM. X-Ray demonstrates joint effusion w/widening. A. osteochondritis dissecans B. slipped capital femoral epiphysis C. septic hip arthritis D. Legg-Calvé-Perthes disease

C. Meckel diverticulum Meckel diverticulum is prevalent in 2% of the population, has a 2:1 male:female predominance, and is usually located 2 ft from the ileocecal valve. The most common clinical presentations are bleeding, intestinal obstruction, and inflammation. Bright red or maroon bleeding is the most frequent complication in children younger than 2 years of age

A 6-yo girl presents to the ED with abdominal distension of 1-day duration. She has not had a bowel movement or passed flatus in 72 hours. Examination reveals markedly diminished bowel sounds with tympany to percussion. She has also passed bloody mucus from her rectum. There is no evidence of hernia, and surgical history is negative. Which of the following is the most likely diagnosis? A. Regional enteritis B. Pyloric stenosis C. Meckel diverticulum D. Acute appendicitis

(E) A serum calcium level of 6.5 mg/dL on the second hospital day is a bad prognostic sign.

A 60-year-old alcoholic is admitted to the hospital with a diagnosis of acute pancreatitis. Upon admission, his white blood cell (WBC) count is 21,000. His lipase is 500, blood glucose is 180 mg/dL, lactate dehydrogenase (LDH) is 400 IU/L, and aspartate aminotransferase (AST) is 240 IU/dL. Which of the following is TRUE? (A) This patient is expected to have a mortality rate of less than 5%. (B) The patient's lipase level is an important indication of prognosis. (C) This patient requires immediate surgery. (D) A venous blood gas would be helpful in assessing the severity of illness in this patient. (E) A serum calcium level of 6.5 mg/dL on the second hospital day is a bad prognostic sign.

(A) The hernia is more likely to be direct than indirect. Hernias, which present in adult life are most often direct and aquired, rather than indirect. They protrude through the transversalis fascia, which forms the medial half of the posterior wall of the inguinal canal and is located medial to the deep inguinal ring and deep epigastric vessels. Strangulation of direct inguinal herniae is uncom- mon, probably because the neck of the sac tends to be wide, rather than narrow and constricting

A 60-year-old male presents with an inguinal hernia of recent onset. Which of the following statements are TRUE? (A) The hernia is more likely to be direct than indirect. (B) Presents through the posterior wall of the inguinal canal, lateral to the deep inguinal ring. (C) Is covered anteriorly by the transversalis fascia. (D) Is more likely than a femoral hernia to strangulate. (E) The sac is congenital.

(A) Angiography to look for angiodysplasia A common cause of lower GI bleeding that is recurrent and painless is angiodysplasia of the colon. In the absence of diverticula or hem- orrhoids, the suspicion is even higher for these lesions. Peptic ulcer and Meckel's diverticu- lum can cause predominantly lower GI bleed- ing. However, the bleeding is usually in the form of melena rather than bright red.

A 60-year-old man complains of recurrent attacks of painless rectal bleeding. Colonoscopy reveals normal mucosa between the cecum and the anal verge. What is the most helpful test to determine the cause of bleeding? (A) Angiography to look for angiodysplasia (B) Technetium scan for Meckel's diverticulum (C) Upper GI endoscopy for peptic ulcer (D) Small-bowel series for tumor (E) Ultrasound for abdominal aortic aneurysm

(C) Heart The heart is the origin of about 90% of lower extremity emboli.

A 60-year-old man with a history of atrial fibrilla- tion is found to have a cyanotic, cold right lower extremity. The embolus is most probably originating from which of the following? (A) An atherosclerotic plaque (B) An abdominal aortic aneurysm (C) Heart (D) Lungs (E) Paradoxical embolus

(E) CT with intravenous and oral contrast A CT scan with intravenous and oral con- trast can best rule out possible retroperitoneal injury caused by a stab wound.

A 60-year-old woman presents with a stab wound to the back just above the iliac crest. She is in stable condition. (A) Peritoneal lavage (B) Wound exploration (C) Sonogram (D) Paracentesis (E) CT with intravenous and oral contrast (F) IVP (G) Exploratory laparotomy (I) Angiogram

B. cessation of smoking

A 62-year-old male who has not had medical care for many years was seen 2 days ago at the ED for chest pain that was diagnosed as angina pectoris. At follow-up today, he admits to multiple poor health habits: cigarette smoking since age 12, consumption of 3-4 beers daily, a diet of fast food 3 times a day, 5-7 cups of regular coffee per day, and virtually no exercise.What is the single most important lifestyle modification that he should make? A. abstinence from alcohol B. cessation of smoking C. dietary modifications D. increasing exercise

E. Referral to a gynecologist for endometrial biopsy

A 64 yr old female presents to ED with vaginal bleeding for 4 days. She denies abd pain and describes bleeding as light. VS are normal and exam confirms trace blood in the vaginal vault. Her hematocrit is stable and US does not show a cause for her symptoms. Which of the following is the most appropriate next step in management? A. Abdominal CT scan B. f/u with her PMD for routine physical exam C. No further care is warranted D. Progesterone 10 mg/day for 10 days E. Referral to a gynecologist for endometrial biopsy

(C) He should undergo left hemicolectomy.

A 64-year-old train conductor is diagnosed as having carcinoma confined to the descending colon. Before operation, what should he be told? (A) He will most likely require a colostomy. (B) He should have the cancer excised by cautery. (C) He should undergo left hemicolectomy. (D) Radiotherapy is the treatment of choice. (E) 40% of colorectal cancer involves the colon.

(C) Asymptomatic aneurysm 5.5 cm

A 65-year-old female on her routine examina- tion was noted to have a pulsatile abdominal mass. She has been otherwise healthy with history of hypertension with no other history, except family history of father dying of rup-tured AAA. What are the acceptable reasons to operate on abdominal aortic aneurysms in 65-year-old female with 5-cm infrarenal aneurysm? (A) Presence of aneurysm (B) Aneurysm with intramural thrombus (C) Asymptomatic aneurysm 5.5 cm (D) Associated 2-cm iliac aneurysm (E) Patient with splenic artery aneurysm 1.5 cm

(A) The sac is formed by an unobliterated processus vaginalis

A 65-year-old female requires emergency surgery for a strangulated inguinal hernia. Which of the following is correct? (A) The sac is formed by an unobliterated processus vaginalis. (B) The hernia is direct rather than indirect. (C) Such herniae never contain small intestine. (D) Strangulation never results in bowel ischemia and gangrene requiring resection. (E) Indirect inguinal herniae are never found in female patients.

(C) Intramural hematoma of the proximal small bowel

A 67-yo woman is hospitalized b/c of abdominal pain and persistent copious vomiting for 24 hours. 2 weeks ago, she was hospitalized for a-fib; after cardioversion to a NSR, she began treatment with warfarin. Yesterday at a follow-up visit, her INR was 6, and her medication was d/c. She takes no other meds. Her temp is 37°C (98.6 F), pulse is 120/min and regular, resp are 20/min, and BP is 100/78 mm Hg. The abdomen is distended and moderately tender; there is voluntary guarding in the epigastrium. There are no masses, organomegaly, or obvious hernias. Rectal exam shows no abnormalities. Test of the stool for occult blood is negative. Her hemoglobin concentration has decreased from 13 g/dL yesterday to 7.8 g/dL today. An ECG shows a NSR. Which of the following is the most likely explanation for this patient's abdominal symptoms? (A) Internal small-bowel herniation (B) Intestinal ischemia from a cardiac embolus (C) Intramural hematoma of the proximal small bowel (D) Intussusception of the small bowel (E) Malrotation of the small bowel

B) Laser vaporization

A 68 year old complains of vulvar pruritis. Colposcopy reveals 3 multifocal <1cm lesions. Biopsy reveals vulvar intraepithelial neoplasia (VIN) 3. The best treatment is: A) Skinning vulvectomy B) Laser vaporization C) 5-FU D) Radical vulvectomy E) Wide local excision

(A) Small-bowel intestinal obstruction due to adhesions

A 68-year-old man presents with crampy abdominal pain and distention with vomiting. Findings on physical examination are positive for healed abdominal scars. X-rays reveal multiple gas fluid levels. The WBC count is 12,000. What is the most likely diagnosis? (A) Small-bowel intestinal obstruction due to adhesions (B) Hernia (C) Appendicitis (D) Inflammatory bowel disease (E) Gallstones and ascites

(C) Zinc deficiency Both zinc and vitamin C (ascarbate) deficiency, impair wound healing. Vitamin A deficiency is also implicated in would heal- ing and supplemental Vitamin A has been shown in experimental studies to prevent radi- ation included defects in wound healing. Incision through the same abdominal wall scar incision actually promotes wound healing, because the initial lag interval after creation of the wound is avoided (unless the whole scar of the incision is removed). Increase in local oxygen tension actually promotes wound healing.

A 68-year-old retired female plastic surgeon underwent laparotomy through a midline abdominal incision. Intestinal infarction was found and a distal 60% small-bowel resection was performed with ileocecal anastomosis. She was placed on hyperalimentation. Seven days after the operation, she underwent a second operation through the same incision. Wound healing is further impaired by which of the following? (A) Incision through the same abdominal wall scar (B) Vitamin A administration (C) Zinc deficiency (D) Increased local oxygen tension (E) Incision through new area of abdominal wall

(C) 6-8 months

A 7-month-old infant presents for well-child care and the physician notes a well-developed Moro reflex, palmar grasp reflex, and tonic neck reflexes on examination. The persistence of neonatal reflexes is an indicator of developmental delay, he tells the mother of the infant. At which age should the Moro reflex typically disappear? (A) 3 months (B) 4 months (C) 6-8 months (D) 12-16 months (E) 17-20 months

(D) Symptom occurrence in more than one environment

A 7-year-old girl comes to her pediatrician's office because her teacher is concerned that she may have attention-deficit hyperactivity dis- order (ADHD). Hallmarks of ADHD include which of the following? (A) Often quiet demeanor (B) History of seizures at birth (C) History of recurrent ear infections (D) Symptom occurrence in more than one environment (E) Does not appear to be easily distracted

(D) urine analysis

A 7-year-old girl develops secondary nocturnal enuresis. What is the next best study? (A) renal ultrasound (B)voiding cystourethrogram (C) abdominal radiograph (D) urine analysis (E) creatinine clearance

(A) Direct inguinal hernia

A 70-year-old cigarette smoker presents with a right inguinal mass that has enlarged and has caused discomfort in recent months. He com- plains of recent difficulty with micturition and nocturia. The swelling, which does not extend to the scrotum, reduces when resting. What is the likely diagnosis? (A) Direct inguinal hernia (B) Strangulated indirect inguinal hernia (C) Hydrocele (D) Aneurysm of the femoral artery (E) Cyst of the cord

(C) HIDA scan A HIDA scan will show excretion of the radiolabeled isotope into the biliary tree, but there will be no flow into the duodenum, indi- cating that the biliary-enteric anastomosis is occluded. If an upper GI study with barium is performed, visualization of the common bile duct would indicate patency of the choledochoduodenal anastomosis.

A 70-year-old male underwent a choledochoduodenostomy for multiple common duct stones. The patient now presents with RUQ abdominal pain. What should be the initial test (least invasive with the best yield) to determine patency of the choledochoduodenostomy? (A) ERCP (B) Percutaneous transhepatic cholangiogram (PTC) (C) HIDA scan (D) CT scan of the abdomen (E) Ultrasound of the abdomen

(D) Aspirin

A 75-year-old man is found to have prolonged bleeding from intravenous puncture sites. Platelet aggregation is inhibited by which of the following? (A) Adenosine diphosphate (ADP) (B) Calcium (C) Magnesium (D) Aspirin (E) Serotonin

C) Insertion of a nasogastric tube

A 78-year-old woman undergoes an uncomplicated minor surgical procedure under local anesthesia. At the completion of the operation, she suddenly develops pallor, sweating, bradycardia, hypotension, abdominal pain, and gastic distension. What is the next stem in management? (A) Rapid infusion of 3 L of Ringer's lactate (B) Digoxin (C) Insertion of a nasogastric tube (D) Morphine (E) Neostigmine

D) The omentum and peritoneal cavity appear to be less efficient in localizing the disease in these age groups.

A 79-year-old man has had abdominal pain for 4 days. An operation is performed, and a gangrenous appendix is removed. The stump is inverted. Why does acute appendicitis in elderly patients and in children have a worse prognosis? (A) The appendix is retrocecal. (B) The appendix is in the preileal position. (C) The appendix is in the pelvic position. (D) The omentum and peritoneal cavity appear to be less efficient in localizing the disease in these age groups. (E) The appendix is longer in these age groups.

D. Femoral neck fracture

A 88-yo female is found lying in a supine position after sustaining a fall in her house. Her leg is shortened, abducted and externally rotated, and she is complaining of pain to the right leg that radiates to the knee. Based on the history of the patient, what type of injury has this patient sustained? A.Non-displaced pelvic fracture B. Hip dislocation C. Femoral shaft fracture D. Femoral neck fracture E. Distal femur fracture

(A) decreased serum C3

A 9-year-old has hematuria and an increased serologic titer to antistreptolysin O (ASO). What will be the most likely finding? (A) decreased serum C3 (B) IgA deposits in kidney biopsy (C) decreased serum albumin (D) decreased urinary protein/creatine ratio (E) hypercalciuria

(D) child abuse Child abuse and neglect are correlated with delayed language, particularly the ability to convey emotional states.

A group of student nurses is learning about childrens' language development. Their instruc- tor explains that language is a critical barometer of both cognitive and emotional development. With which of the following is speech delay most closely associated? (A) 22q11 deletion syndrome (B) spina bifida (C) diabetes (D) child abuse (E) asthma

(C) 12 g/dL

A hemoglobin and hematocrit is obtained from a 12-month-old who came to his pediatrician's office for a checkup. Which of the following best describes the normal (average) hemoglobin concentration at 1 year of age? (A) 17 g/dL (B) 15 g/dL (C) 12 g/dL (D) 10 g/dL (E) 8 g/dL

(D) Penicillin penicillin, because administration of this medication in children with sickle cell disease who are younger than 5 years of age decreases the risk of serious bacterial infections such as pneumonia.

A male neonate is delivered vaginally at term, and neonatal examination and testing confirms the diagnosis of sickle cell disease. On the basis of this finding, the most appropriate initial step is administration of which of the following? (A) Erythromycin (B) Hydroxyurea (C) Oxygen (D) Penicillin (E) Pneumococcal and meningococcal vaccines

(B) Pressure-gradient stockings

A middle-age woman has right leg and foot nonpitting edema associated with dermatitis and hyperpigmentation. The diagnosis of chronic venous insufficiency is made. What is the treatment of choice? (A) Vein stripping (B) Pressure-gradient stockings (C) Skin grafting (D) Perforator vein ligation (E) Valvuloplasty

(B) Ultrasound Although aortography, CT, and MRI can all establish the diagnosis of abdominal aortic aneurysm, ultrasound remains the best screen- ing test.

A middle-aged man is found to have a small pulsating mass at the level of the umbilicus during a routine abdominal examination. What is the best initial test to establish the diagnosis? (A) Aortography (B) Ultrasound (C) Computed tomography (CT) (D) Magnetic resonance imaging (MRI) (E) Plain films of the abdomen

(A) 9-12months

A mother brings her 6-month-old son in for a checkup. She is concerned that when she is playing with him and puts a toy behind her back, he does not try to find it. Object permanence, the understanding that objects continue to exist even when not seen, is a major milestone that occurs closest to which of the following ages? (A) 9-12months (B) 15months (C) 18months (D) 24months (E) 30months

(C) 40 mg/dL

A normal newborn is screened for hypoglycemia after birth as per the normal nursery protocol at your hospital. In the asymptomatic term neonate, evaluation and treatment of hypoglycemia should be initiated when the glucose level is at or below which of the following levels? (A) 10 mg/dL (B) 20 mg/dL (C) 40 mg/dL (D) 60 mg/dL (E) 80 mg/dL

(E) Stable, individual modes of responding to the environment.

A pediatric psychologist is discussing the concept of temperament with a group of residents. Which of the following best describes the term temperament? (A) A parent's response to a tantrum. (B) The emotional bond that a child feels with his parent. (C) The acts of independence that a child demonstrates as she enters toddlerhood. (D) A child's emotional clinginess to a parent when faced with an unfamiliar situation. (E) Stable, individual modes of responding to the environment.

d) there is a high likelihood of liver involvement if the CEA level is high (greater than 100 ng/mL)

A right hemicolectomy is performed on a 57-year-old woman with adenocarcinoma who had a preoperative elevation of carcinoembryonic antigen (CEA) to 144. After falling to normal levels postoperatively, her most recent (24-month) follow-up level was 86. Correct statements regarding CEA and colorectal tumors include which of the following? a) elevated CEA is indicative of a tumor of gastrointestinal origin b) a low CEA level after resection of a colon tumor is a poor marker of disease control c) 90% of colorectal tumors produce CEA d) there is a high likelihood of liver involvement if the CEA level is high (greater than 100 ng/mL) e) CEA levels are unusually low in cigarette smokers

D) Increase serum testosterone *Androgen insensitivity syndrome*: a person who is genetically male (who has one X and one Y chromosome) is resistant to male hormones (called androgens). As a result, the person has some or all of the physical traits of a woman, but the genetic makeup of a man. **Testosterone will be elevated in this! *Malarian Agenesis*: embryologic growth failure of mullerian duct, resulting in agenesis or underdeveloped vagina and/or uterus; diagnosed by US

A teen has primary amenorrhea, normal external genitalia, but a blind ending vagina. What would be the most helpful to differentiate androgen insensitivity syndrome from mullerian agenesis? A) Absent uterus B) Café au lait spots C) Urinary tract abnormalities D) Increase serum testosterone E) Normal LH

(D) This amount of crying is normal for this age. Crying gradually increases during the first 6-12 weeks of age because it is the main modal- ity by which infants express responses to stim- uli, both aversive and nonaversive. Crying can be a response to a variety of stimuli, including hunger, a wet diaper, fear, fatigue, and over- stimulation

A tired-appearing mother of a 2-month-old boy states he is feeding and sleeping well but cries for a total of 3 hours each day. The baby's examination is normal. Which of the following statements about crying is correct? (A) Crying increases through the entire first year of life. (B) Crying usually is a result of hunger. (C) Any baby crying 3 hours per day, even in light of a normal examination, warrants a medical workup. (D) This amount of crying is normal for this age. (E) This degree of crying warrants an immediate skeletal survey.

(B) 30 g/day or 1 oz/day Newborns may lose up to 10% of their birth weight in the first few days of life, but with normal nutrition birth weight is regained in approximately 10 days. The infant subse- quently gains approximately 30 g/day for the first several months

After 2 weeks of age, a term neonate will gain an average of which of the following increments of weight? (A) 15 g/day or 1/2 oz/day (B) 30 g/day or 1 oz/day (C) 45 g/day or 11/2 oz/day (D) 60 g/day or 2 oz/day (E) 120 g/day or 4 oz/day

C. Oral antibiotics

After a thorough history and physical examination, what is the most appropriate emergency department management for uncomplicated diverticulitis in a middle-aged patient? A. Abdominal and pelvic CT scanning B. Lower gastrointestinal barium contrast study C. Oral antibiotics D. Parenteral antibiotics

(A) homicide and legal intervention

Among adolescents of 15-19 years of age, motor vehicle accidents cause the greatest number of deaths each year. Which of the following is the second leading cause of death in this age group? (A) homicide and legal intervention (B) suicide (C) heart disease (D) cancer (E) cystic fibrosis

(A) Aspiration of the right chest cavity In a patient presenting with a chest wound in shock, the priorities are airway, breathing, and circulation. Thus, aspiration of the right chest to rule out a tension pneumothorax should be per- formed first. Aspiration of the pericardium does not definitively rule out cardiac injury; a peri- cardial window provides both diagnosis and decompression. An echocardiogram is not indicated in an unstable patient.

An 18-year-old man is brought to the emergency department with a stab wound just to the right of the sternum in the sixth intercostal space. His blood pressure is 80 mm Hg. Faint heart sounds and pulsus paradoxus are noted. Auscultation of the right chest reveals decreased breath sounds. The initial management of this patient should be which of the following? (A) Aspiration of the right chest cavity (B) Aspiration of the pericardium (C) Echocardiogram (D) Pericardial window (E) Insertion of central venous access line

(C) Pleurocentesis The immediate treatment is the closure of the hole by any means available. Sucking chest wounds allow shift of the mediastinum to the opposite side. Thoracotomy is not usually required. Laparotomy is indicated for a gun- shot wound below the fourth intercostal space, but it should follow respiratory stabilization. A chest tube will be required, following closure of the sucking wound, to prevent a tension pneumothorax.

An 18-year-old man presents to the emergency department with a gunshot wound to the left chest in the anterior axillary line in the seventh intercostal space. A rushing sound is audible during inspiration. Immediate management is which of the following? (A) Exploratory laparotomy (B) Exploratory thoracotomy (C) Pleurocentesis (D) Closure of the hole with sterile dressing (E) Insertion of chest tube

(C) poison ivy dermatitis

An 8-year-old child develops an intensely pruritic rash on the legs only. There are patches of erythematous papules and vesicles and several streaks of erythematous vesiculation. The child is afebrile and otherwise well. The most likely diagnosis is (A) eczema (B) Henoch-Schönlein purpura (C) poison ivy dermatitis (D) scabies (E) varicella

D. Reye's syndrome Although rare, Reye's syndrome is associated with viral infections, salicylate use during illness, and metabolic disorders. Illness is associated with liver fat deposition and degeneration, intractable vomiting, and mental status changes, which may progress to seizures, delirium, and coma. Cerebral edema contributes to these changes and other neurologic findings.

An 8-yo girl is rushed to the ED by her parents because she has become delirious. The child was diagnosed with the flu 3 days prior. Her parents say that she had begun vomiting yesterday, almost nonstop, and has not been able to hold down fluids. They also note that she has been breathing rapidly. Your exam reveals a tachypneic, disoriented female with hyperreflexia, a positive babinski reflex, and liver enlargement. CSF analysis reveals a normal protein and cell count. What is the most likely diagnosis? A. Bacterial meningitis B. Guillain Barre syndrome C. Measles encephalitis D. Reye's syndrome E. Viral meningitis

c) Colon resection and proximal colostomy

An 80-year-old man is admitted to the hospital complaining of nausea, abdominal pain, distention, and diarrhea. A cautiously performed transanal contrast study reveals an apple core configuration in the rectosigmoid area. Which of the following is the most appropriate management at this time? a) Colonoscopic decompression and rectal tube placement b) Saline enemas and digital disimpaction of fecal matter from the rectum c) Colon resection and proximal colostomy d) Oral administration of metronidazole and checking a Clostridium difficile titer e) Evaluation of an electrocardiogram and obtaining an angiogram to evaluate for colonic mesenteric ischemia

(D) Richter's hernia In a Richter hernia, only part of the cir- cumference of the bowel wall has become trapped in the hernia sac, and normal bowel movements may still occur. In the presence of a reducible groin hernia, it is important on clinic examination to be certain that other pathologic conditions are not overlooked.

An 80-year-old woman with a known history of femoral hernia is admitted to the hospital because of strangulation of the hernia. There is a tender swelling in the right femoral region immediately below and lateral to the pubic tubercle. She has had multiple bowel move- ments without relief of symptoms. What is the most likely diagnosis? (A) Lymphadenitis (B) Diverticulitis (C) Volvulus (D) Richter's hernia (E) Gastroenteritis

(A) arbovirus

An 8yo child presents to the ED with a 3-day history of fever and new altered mental status. Her parents report that they have been camping several times and she has sustained a lot of mosquito bites. Mosquitoes are recognized as vectors in the transmission of encephalitis of which of the following viruses? (A) arbovirus (B) coxsackievirus (C) enterovirus (D) influenza virus (E) mumps virus

C. prednisone 2.4 mg/kg/24 hours × 2 weeks with idiopathic thrombocytopenic purpura, treatment options should be initiated when platelet counts fall below 20,000, regardless ofactive bleeding or not. Without active bleeding the treatment options include prednisone 2-4 mg/kg/24 hours for 2 weeks; IV immunoglobulin 1 g/kg/24 hours for 1 to 2 days, or IV anti-D 50-75 μg/kg/dose for Rh-positive patients. Splenectomy is indicated for life-threatening bleeding. There is currently no indication for platelet transfusion

An Rh-negative, 5-year-old male child presents with acute onset of petechiae and purpura after an acute viral illness. In addition, he has episodes of epistaxis. Which of the following is a treatment option if his platelet count falls below 20,000/mm 3 , but he is not actively bleeding? A. platelet transfusions B. IV anti-D (WinRho SD) 50-70 mg/kg/dose C. prednisone 2.4 mg/kg/24 hours × 2 weeks D. splenectomy

(B) cystic fibrosis

An infant is diagnosed with a given disorder below. Which of these poses the greatest recurrence risk for this patient's future siblings? (A) Hirschsprung disease (B) cystic fibrosis (C) ventricular septal defect (D) trisomy 21 (E) trisomy 13

(B) Obtain a CT scan with intravenous contrast if able.

An older gentleman with a hx of alcoholism and chronic pancreatitis presents with pain radiating to the back. He states the pain is much worse than before. He has a mild fever. His WBC count is normal, but you note that his hemoglobin level is 8.5 mg/dL. You look at the lab values in his medical record and note that it was 10.5 on a prior hospitalization. Lab values, including liver function tests, amylase, and lipase, are normal. What is your next step? (A) Send the gentleman home because the lipase is normal. (B) Obtain a CT scan with intravenous contrast if able. (C) Obtain an outpatient gastrointestinal consultation. (D) Obtain an abdominal ultrasound. (E) Repeat the labs because there may be a mistake.

(D) 6months

At a 4-month well child visit, a mother is distressed because her baby's grandmother believes she should have started feeding her baby solid food at 2 months of age. It is generally recommended that beikost (infant foods other than milk) be introduced into the infant's diet at about what age? (A) 3weeks (B) 6weeks (C) 3months (D) 6months (E) 1year

C. Surgical evaluation The patient is high risk, as she is premenopausal, has a family history of cancer, and the mass is large. Therefore, surgical evaluation should be undertaken. CA 125 can be negative in early disease, and pelvic US and CT are not sensitive enough. Repeat examination should be reserved for low risk women with smaller ovarian masses.

At her routine annual pelvic exam, a 39-yo female presents to the clinic complaining of pelvic pressure and bloating for several months. She is a G3P2 who delivered vaginally. She is a nonsmoker. Her maternal aunt had a history of ovarian cancer. Her pelvic exam reveals an 8-cm ovarian mass in the right adnexal area. What is the most appropriate evaluation of the ovarian mass? A. Pelvic ultrasound B. Pelvic CT scan C. Surgical evaluation D. CA 125 level E. Repeat pelvic exam 1 month

(B) In the majority of cases, the abuser is well known to the child.

Concerning child sexual abuse, which of the following is correct? (A) Boys and girls are at equal risk. (B) In the majority of cases, the abuser is well known to the child. (C) Physical contact is necessary to fulfill diagnostic criteria for child sexual abuse. (D) An estimated 10% of children are sexually abused each year in the United States. (E) There must be evidence of sexual inter- course for successful prosecution.

(B) head control, hands together in midline, rolling over, pulls to stand

First-time parents of a 1-month-old are anx- ious to hear what they can expect in the next few months. Which of the following best describes the usual sequence for attainment of gross motor milestones in a young infant? (A) head control, rolling over, hands together in midline, sits without support (B) head control, hands together in midline, rolling over, pulls to stand (C) rolls over, sits without support, hands together in midline, pulls to stand (D) sits without support, hands together in midline, pulls to stand, walks along (E) pulls to stand, walks along table, sits without support, hands together at midline

(A) Mesenteric embolus

Four days after suffering MI, a 78-year-old woman suddenly develops severe diffuse abdominal pain. Her ECG shows atrial fib. On examination, the abdomen is soft, minimally tender, and slightly distended. Hyperactive bowel sounds are present. What is the most likely diagnosis? (A) Mesenteric embolus (B) Nonocclusive ischemic disease (C) Perforated peptic ulcer (D) Congestive heart failure (CHF) (E) Digoxin toxicity

(D) trisomy 21 (Down syndrome)

Hypotonia and hyperextensible joints, clinodactyly, Brushfield spots, duodenal atresia, flattening of the occiput are all characteristics of which syndrome? (A) trisomy 13 (Patau syndrome) (B) trisomy 18 (Edwards syndrome) (C) 5p deletion (cri-du-chat syndrome) (D) trisomy 21 (Down syndrome) (E) 45,X or XO (Turner syndrome)

(B) Ultrasound Many PANCE questions concerning congenital hip dysplasia in the newborn focus on the Barlow and Ortolani maneuvers. The best imaging study for the newborn is the ultrasound. In the first few months of life, much of the joint may not be well visualized on standard radiologic imaging.

In addition to physical examination, which one of the following is most useful in diagnosing congenital hip dysplasia in the newborn? (A) Radiograph (B) Ultrasound (C) CT scan (D) MRI (E) Bone scan

(C) name two or three colors The gross motor tasks described (walking downstairs with alternate feet, throwing overhand, broad jump) are accomplished at 3-4 years of age

In coordinating pediatric residents to perform head start (early childhood development) physicals and developmental screenings on a group of preschool children, it is mentioned that a child who can walk downstairs alternating his or her feet, do a broad jump, and throw a ball overhand, also would be expected to do which of the following? (A) add five and five (B) identify three or four coins (C) name two or three colors (D) multiply three times three (E) write his or her name

(D) folate deficiency

Infants fed exclusively goat milk are susceptible to which of the following? (A) vitamin A deficiency (B) vitamin B6 deficiency (C) vitamin E deficiency (D) folate deficiency (E) thiamine deficiency

(D) Saline fluid replacement with appropriate potassium administration

Initial treatment for hypochloremic metabolic alkalosis should include which of the following? (A) Administration of 10% dextrose (D10W) in one-third saline solution IV (B) Antiemetic (C) Hemodialysis to correct azotemia (D) Saline fluid replacement with appropriate potassium administration (E) Ringer's lactate solution

(B) trisomy 18 (Edwards syndrome)

Low-set ears, nail hypoplasia, "rocker-bottom" feet, growth retardation, severe mental retardation (A) trisomy 13 (Patau syndrome) (B) trisomy 18 (Edwards syndrome) (C) 5p deletion (cri-du-chat syndrome) (D) trisomy 21 (Down syndrome) (E) 45,X or XO (Turner syndrome)

(A) lethargy and gastrointestinal irritation The clinical manifestations of iron poison- ing have been organized into four stages. The first stage is characterized by gastrointestinal (vomiting, diarrhea, abdominal pain, and gas- trointestinal bleeding) and neurologic (lethargy or coma) signs. This is followed by a second stage of deceptive quiescence, of up to 48 hours, and then a third stage characterized by shock and metabolic acidosis, with or without evidence of hepatic injury. Leukocytosis is common. Late sequelae (stage four) include pyloric or antral stenosis and hepatic cirrhosis.

Manifestations of the first stage of severe acute iron poisoning include (A) lethargy and gastrointestinal irritation (B) metabolic alkalosis and hypertension (C) hemolysis and neutropenia (D) renal, hepatic, and cardiac failure (E) hypoglycemia and hepatic injury

(A) salicylates

Overdose of which of the following is most likely to be complicated by hypoglycemia? (A) salicylates (B) lead (C) tricyclicantidepressants (D) opioids (E) organophosphates

(C) of normal height and weight Children with constitutional growth delay are initially short, but have a longer than normal period of growth and a later than normal adolescent growth spurt. Although there are some exceptions, ultimate height usu- ally is normal, and ultimate weight is normal and proportionate to height.

Parents bring a 10-year-old boy in for evaluation of short stature. The patient is worried about his eventual adult height. Children with consti- tutional growth delay (without endocrine abnor- mality) generally can expect to ultimately achieve which of the following growth descriptions? (A) very short and obese (B) short but of proportionate weight (C) of normal height and weight (D) very tall but of proportionate weight (E) very tall and obese

A. Drugs that stimulate ovulation are a risk factor for ovarian torsion Risk factors for developing ovarian torsion include pregnancy, the presence of a large ovarian cyst or mass, and chemical induction of ovulation. US is often the first imaging modality to evaluate torsion but should not be relied upon to r/o the dx when clinical suspicion is high.

Regarding ovarian torsion, which of the following statements is correct? A. Drugs that stimulate ovulation are a risk factor for ovarian torsion. B. If a patient has vague, bilateral lower abd pain, ovarian torsion is extremely unlikely C. Ovarian torsion most commonly occurs on the left side D. Pelvic US is highly reliable for the evaluation of ovarian torsion E. Pregnancy is not a risk factor for ovarian torsion

(A) "me do it"

The cognitive developmental thrust of the 2-year- old is best expressed by which of the following phrases? (A) "me do it" (B) "show me how" (C) "that can't be right" (D) "why" (E) "you do it"

(E) Preschool-aged children may demonstrate regressive behavior. Adverse effects may vary by age and gender, and include regressive behavior, depression, anger, anxiety, declining academic performance, and other externalizing and internalizing behaviors. Boys in general appear to have a more difficult initial adjustment than do girls. However, as they reach young adulthood, girls may experience a "sleeper" effect; though their initial adjustment occurred years before, they may undergo anxiety about male-female relationships. In a longitudinal study, nearly 40% of children were doing poorly (academic problems, behavior problems at school and/or home) 5 years after divorce

The father of 2- and 8-year-old siblings calls their pediatrician to discuss how he and the childrens' mother can prepare them for the par- ents' upcoming divorce. Which of the following statements is true regarding children of divorce? (A) The emotional availability of one or both parents is often heightened. (B) On average, girls have a more difficult time adjusting initially to divorce than boys. (C) As boys reach young adulthood, they may experience anxiety about male- female relationships (D) The majority of children have adjusted to the altered family circumstance by the second year following the divorce. (E) Preschool-aged children may demonstrate regressive behavior.

D. Encephalopathy

The hypertensive emergency that is most easily reversible with pharmaceutical management is: A. Acute coronary syndrome B. Aortic dissection C. Eclampsia/preeclampsia D. Encephalopathy E. Intracranial hemorrhage

(D) Recurrent otitis media

The inpatient pediatric team is discussing the causes of failure to thrive (FTT) as they evaluate a 9-month-old infant with weight and length below the 5th percentile. Which of the following should be included in the differential diagnosis of infant who is not thriving? (A) Small atrial septal defect (B) Acute bacterial meningitis (C) Cystic fibrosis (D) Recurrent otitis media (E) Mild intermittent asthma

B. minimal change disease

The most common cause of nephrotic syndrome in children is A. post-streptococcal glomerulonephritis B. minimal change disease C. diabetes mellitus D. NSAIDs E. polycystic kidney disease

C. Helical CT chest scan

The most specific diagnostic test that can be obtained most rapidly for an emergency department patient to make the initial diagnosis of aortic dissection is: A. 12-lead ECG B. Aortic angiography C. Helical CT chest scan D.Portable chest x-ray E. Transesophageal echocardiography

(C) 8-9 months The advent of object constancy corresponds with qualitative changes in social and commu- nicative development. The infant looks back and forth between an approaching stranger and a parent, as if to contrast known from unknown, and may cling or cry anxiously. This occurs around 8-9 months of age

The mother of 9-month-old, who had a normal sleeping pattern in the past, reports he has started awakening and crying once or twice a night. Her primary care physician suggests that this may be a manifestation of separation anxiety. At which of the following ages does separation anxiety usually first manifest? (A) 1 week (B) 3-4 months (C) 8-9 months (D) 2-3 years (E) 5-6 years

(E) an emotional or psychiatric problem *Declining scholastic grades are always worrisome.*

The mother of a 16-year-old girl reports that she has noticed declining scholastic grades, refusal to participate in family activities, and an unwillingness to communicate with either parents or peers. These findings in an adolescent usually indicate which of the following? (A) a normal stage of development (B) a normal response to peer pressure (C) a normal reaction to overprotective parents (D) a transient phase of ambivalence (E) an emotional or psychiatric problem

(B) usually appear at the end of the first year Tantrums normally appear at the end of the first year of life and peak at 2-4 years.

The mother of a 2-year-old girl brings her to her pediatrician's office for a checkup. When asked about discipline concerns, the mother expresses worry because the child gets mad and throws herself on the ground screaming. Which of the following statements regarding temper tantrums is correct? (A) most often indicate a serious psychoso- cial problem (B) usually appear at the end of the first year (C) peak prevalence is between 4 and 6 years (D) routinely occur 8-10 times per day (E) usually last between 30 and 45 minutes

(A) feeds self, helps to undress, washes hands, domestic role-playing

The mother of a 30-month-old brings the primary care provider a note from his parents as teachers (early childhood development) specialist. The PAT specialist is concerned by the child's lack of self-feeding skills. Which of the following social development sequences do toddlers follow? (A) feeds self, helps to undress, washes hands, domestic role-playing (B) handles spoon well, plays in parallel, dresses and undresses, washes hands (C) plays in parallel, feeds self, dresses and undresses, washes hands (D) plays in parallel, dresses and undresses, washes hands, feeds self (E) washes hands, dresses herself, plays in parallel, handles spoon well

(A) hearing and visual evaluation

The mother of a 6-year-old in first grade reports that he is not keeping up with the other kids in any subject and wants testing to be performed. Evaluation of academic failure at school typically includes which of the following? (A) hearing and visual evaluation (B) personality testing (C) magnetic resonance imaging (MRI) of head (D) home visit (E) computed tomography (CT) scan of head

(D) 6 months

The mother of a newborn infant brings her 2-week- old child in for his first checkup in November. The mother has heard reports on the local news of influenza in the community and that young infants are at very high risk if they contract the disease. She asks you if her child can be immu- nized. At what age can a child first receive influenza vaccine? (A) birth (B) 2 months (C) 4 months (D) 6 months (E) 12 months

(D) occurs simultaneously with Tanner stages IV to V pubic hair and breast development

The mother of an 11-yo girl who comes to her checkup notes that her daughter is beginning to develop breast buds. She asks when her daughter might have her first period. Which of the following is a true statement regarding menarche in the adolescent girl? (A) precedes the spurt in linear growth (B) occurs simultaneously with Tanner stage II breast development (C) generally occurs when Tanner stage III breast and pubic hair development have been achieved (D) occurs simultaneously with Tanner stages IV to V pubic hair and breast development (E) generally occurs a year or more after Tanner stage V breast and pubic hair development have been achieved

(A) fully understandable, although mispronunciations and grammatical errors are common

The parents of a 4-year-old boy are concerned because his speech is still not fully understandable. Which of the following best describes a child's conversation at 4 years of age? (A) fully understandable, although mispronunciations and grammatical errors are common (B) fully understandable, with few if any mispronunciations and grammatical errors (C) fully understandable to the parent but not necessarily to others (D) somewhat understandable although garbled and indistinct (E) somewhat understandable, with mostly correct use of nouns and mostly incorrect use of verbs

(C) NICHQ Vanderbilt Assessment Scale

This test evaluates potential ADHD in a 7-year- old. (A) Parents' Evaluations of Developmental Status (PEDS) (B) CAGE questionnaire (C) NICHQ Vanderbilt Assessment Scale (D) Pediatric Symptom Checklist (E) Otoacoustic emission test

(D) Sigmoidoscopy In a patient with abdominal aortic aneurysm resection, the most worrisome complication is inadequate blood supply to the sigmoid colon through the marginal artery. Sigmoid ischemia should be ruled out by sigmoidoscopy. In the clinical picture described, sigmoidoscopy should be the most important test.

Three days after undergoing an operation for an abdominal aortic aneurysm, a patient has moderate fever, abdominal pain, and rectal bleeding. What is the most helpful investigation? (A) Angiography (B) Upper GI endoscopy (C) Abdominal ultrasound (D) Sigmoidoscopy (E) Abdominal CT scan

B) Lactate dehydrogenase

Tumor markers are a helpful adjunct in evaluating a patient with an adnexal mass. However all of the following markers' reliability is altered during pregnancy except: A) HCG B) Lactate dehydrogenase C) Alpha-fetoprotein D) CA-125 E) S Human epididymus protein 4 (HE4)

D) OCPs Sterilization/ Vasectomy/Essure- 99% Copper IUD (Paragard)- 99% Levonorgestrel IUD (Mirena, Skyla) - 99% Etonogestrel Implant (Nexplanon) - 99% Medroxyprogesterone Shots (Depo-Provera) - 97% Combo OCPs - 92% Progesterone only OCPs (mini pill) - 92% NuvaRing - 92% Ortho Evra Patch - 92% Condoms, Diaphragm, Spermicidal jelly - 84-85% Fertility awareness - 75%

Which birth control method has the highest failure rate? A) Sterilization B) Copper IUD C) Vasectomy D) OCPs E) Levonorgestrel IUD

C. Flexion of the hips (McRoberts maneuver) When shoulder dystocia is recognized, the patient's legs should be immediately flexed up to the abdomen with the legs held by the patient known as the McRoberts maneuver. Next, suprapubic pressure should be applied to disimpact the anterior shoulder from the pubic symphysis. Fundal pressure should never be applied!

Which is the first maneuver that should be performed when managing a shoulder dystocia? A. Clockwise rotation of the baby's torso (Wood corkscrew) B. Delivery of the posterior shoulder C. Flexion of the hips (McRoberts maneuver) D. Fundal pressure E. Suprapubic pressure

(D) Hemorrhagic pancreatitis it is ecchymoses and bruising located in the flank areas, is a sign of hemorrhagic pancreatitis.

Which medical condition is associated with Grey-Turner's sign? (A) Acute appendicitis (B) Ulcerative colitis (C) Emphysematous pyelonephritis (D) Hemorrhagic pancreatitis (E) Acute cholecystitis

(D) it is characterized by pruritus and lichenification Atopic dermatitis (eczema) is characterized by exudation, pruritus, and lichenification.

Which of the following best describes atopic dermatitis in children? (A) it tends to spare the face and arms (B) it is frequently associated with uveitis (C) it rarely begins during the first 2 years of life (D) it is characterized by pruritus and lichenification (E) it is associated with elevated serum levels of IgA and IgM and decreased levels of IgE

(B) tonsillitis, rash, and fever in a 5-year-old

Which of the following children is most likely to have group A streptococcal infection? (A) exudative pharyngitis in a 1-year-old (B) tonsillitis, rash, and fever in a 5-year-old (C) cough and pharyngitis in a 15-year-old (D) "slapped cheek" appearance in a 5-year-old (E) fever, congestion, cough, and pharyngitis in a 3-year-old

(D) lethargy, abdominal cramps, constipation, and anemia

Which of the following combinations of signs and symptoms is most suggestive of chronic lead poisoning? (A) ataxia, fever, diarrhea, and polycythemia (B) lethargy, vomiting, hallucinations, and vesicular rash (C) anemia, leukopenia, thrombocytopenia, and hepatomegaly (D) lethargy, abdominal cramps, constipation, and anemia (E) hypertension, rash, cough, and leukocytosis

C. 1,580 mL 100 mL/kg for the first 10 kg of body weight 50 mL/kg for the next 10 kg of body weight 20 mL/kg for the weights above 20 kg

Which of the following daily maintenance fluid requirements is the closest approximation for a 24-kg child who is refusing to eat? A. 1,080 mL B. 1,200 mL C. 1,580 mL D. 2,000 mL

(B) retinal hemorrhages

Which of the following findings would be most suggestive of the form of child abuse referred to as the abusive head trauma syndrome (formally called shaken babt syndrome) (A) ecchymosis over the mastoid area (B) retinal hemorrhages (C) ecchymoses and petechiae over the upper arms and upper trunk (D) circumferential ecchymosis on extremities (E) cervical spine dislocation

B. Hepatitis A Start Hep A at 12months

Which of the following immunizations is not given during the 6 month well visit? A. Hepatitis B B. Hepatitis A C. Rotavirus D. Human influenza B

(B) encopresis Although some cases of encopresis may be secondary to organic causes (spina bifida, Hirschsprung disease), the majority are psy- chologic or behavioral in nature. In contrast, the majority of cases of enuresis are most often due to a disturbance of bladder physiology or sleep mechanism, and generally are not considered to represent an emotional or behavioral disor- der. If a child has been dry, then becomes enuretic (secondary enuresis), emotional fac- tors may be involved in the process. Secondary enuresis may indicate sexual abuse.

Which of the following in an 8-year-old child is most likely to indicate an underlying psychologic or behavioral problem? (A) enuresis (B) encopresis (C) motion illness (D) migraine headache (E) recurrent pharyngitis

E. hypogammaglobulinemia MM is a HYPERgammaglobulinemia - all of the other findings occur in MM

Which of the following is NOT a characteristic feature of multiple myeloma? A. elevated serum calcium B. osteoporosis C. "punched out" osseous lesions D. plasma cell infiltration of bone marrow E. hypogammaglobulinemia

(A) most often occur during the first one- third of the night Night terrors are partial arousals that occur during stage 4 non-REM sleep to near arousal transitions, which typically first present in the preschool-aged child. The child appears awake, but does not respond to surroundings, includ- ing the family members attempting to calm the child. Amnesia of the event is another one of the events distinguishing these from night- mares. Night terrors persist into adolescence in about one-third of the cases.

Which of the following is characteristic of a night terror? (A) most often occur during the first one- third of the night (B) onset usually is during the elementary school years (C) rarely persists until adolescence (D) occurs during REM sleep (E) the event is vividly recalled by the child

(D) galactosemia

Which of the following is most likely to be associated with a cataract in the newborn? (A) maple syrup urine disease (B) glucose-6-phosphate dehydrogenase deficiency (C) phenylketonuria (D) galactosemia (E) propionic acidemia

(E) multicolored bruises along the shins of a 4-year-old child

Which of the following is most suggestive of unintentional (nonabuse) injuries in children? (A) hand print bruise on the face of a child (B) belt marks on the buttocks of a 3-year- old child (C) bruises on upper thighs reported to result from "spanking" (D) fractured femur of a 1-month-old baby from rolling off the bed (E) multicolored bruises along the shins of a 4-year-old child

(A) sudden infant death syndrome (SIDS) Since 1992 the American Academy of Pediatrics has recommended infants be placed on their backs to sleep in order to decrease the risk of SIDS. It has been called "The Back to Sleep" campaign, and has been very successful in helping to decrease SIDS by greater than 40%. Yet SIDS remains the leading cause of death in infants beyond the neonatal period.

Which of the following is the most common cause of deaths in infants under 12 months of age each year in the United States? (A) sudden infant death syndrome (SIDS) (B) RSV bronchiolitis (C) child abuse (D) infantile leukemia (E) congenital heart disease

E. 75-year-old man with 5% BSA superficial partial-thickness burn to his back from a heating pad

Which of the following patients could be safely managed in a community emergency department, thereby avoiding transfer to a burn unit? A. 4-year-old boy with 10% BSA superficial partial-thickness burns to his arms after pulling a pan of boiling water off a stove B. 12-year-old boy with 26% BSA superficial partial-thickness burns to his chest and arms from setting a blanket on fire C. 38-year-old man with 3% BSA full-thickness burn to his hand from a mechanical injury D. 42-year-old woman with diabetes with 5% BSA superficial partial-thickness burns to her feet from scalding bathtub water E. 75-year-old man with 5% BSA superficial partial-thickness burn to his back from a heating pad

(C) runs, goes upstairs alternating feet, rides tricycle, skips

Which of the following sequence best demonstrates the correct attainment of motor milestones? (A) runs, rides tricycle, skips, hops (B) runs, hops, skips, rides tricycle (C) runs, goes upstairs alternating feet, rides tricycle, skips (D) hops, skips, rides tricycle, goes upstairs alternating feet (E) goes upstairs alternating feet, rides tri- cycle, runs, skips

(C) pH 7.25, PCO2 20, HCO3 8 Aspirin poisoning results in a mixed distur- bance of metabolic acidosis and respiratory alka- losis.

Which of the following sets of blood gas values is most compatible with acute aspirin poisoning in a 16-month-old child? (A) pH 7.60, PCO2 40, HCO3 40 (B) pH 7.5, PCO2 40, HCO3 30 (C) pH 7.25, PCO2 20, HCO3 8 (D) pH 7.20, PCO2 45, HCO3 20 (E) pH 7.00, PCO2 35, HCO3 8

(E) Infants under the age of 1 year should ride in restraint devices facing the rear of the car.

Which of the following statements regarding automobile safety for children is correct? (A) Children, beyond the age of 1 year, or 20 lb, may ride either facing the front or rear of the car. (B) Children over 25 lb may use adult-type restraints. (C) A 1-year-old child held in the lap of a seat-belted adult is almost as safe as in an infant restraint device. (D) Safety restraints are not needed for infants less than 3 months of age or 10 lb of body weight. (E) Infants under the age of 1 year should ride in restraint devices facing the rear of the car.

E.Objects longer than 5 cm and wider than 2 cm should be removed before they pass through the stomach

Which of the following statements regarding ingested foreign bodies is correct? A.All children with a suspected foreign body ingestion should undergo x-ray B. Ipecac can be used safely to dislodge a button battery in the esophagus C.Meat tenderizer can be used safely to dissolve an impacted meat bolus D. Most common site of esophageal foreign body entrapment in pediatric patients is the thoracic inlet E.Objects longer than 5 cm and wider than 2 cm should be removed before they pass through the stomach

A. Associated with ST-segment depression in V1

Which of the following statements regarding posterior wall infarction is correct? A. Associated with ST-segment depression in V1 B. ECG shows an inverted T wave in V1 C. ECG shows large S waves in V1 D. Occurs in 5% of all acute MIs E. Results from occlusion of the left anterior descending artery

E. Reduces the work of breathing

Which of the following statements regarding the use of continuous positive airway pressure therapy in pulmonary edema is correct? A. Decreases left ventricular preload and increases afterload B. Decreases mortality rates C. Increases the effective Fio2 delivered D. Lowers intrathoracic pressure E. Reduces the work of breathing

(B) an enlarged heart

Which of the following would be expected in a 6-month-old child with a large ventricular septal defect? (A) cyanosis (B) an enlarged heart (C) a continuous cardiac murmur (D) decreased pulmonary vasculature on roentgenogram (E) evidence of predominantly right ventric- ular hypertrophy on ECG

(A) Cryptorchidism

Which one of the following is a risk factor for developing testicular cancer? (A) Cryptorchidism (B) Varicocele (C) Hydrocele (D) Paraphimosis (E) Hypospadias

(C) The dose of fondaparinux (Arixtra) must be reduced if kidney disease is present.

Which one of the following statements concerning deep venous thrombosis prophylaxis is true? (A) IV heparin administered every 8 hours is acceptable for deep venous thrombosis prophylaxis. (B) Hip surgery for repair of a fracture would be considered a moderate risk for the development of deep venous thrombosis. (C) The dose of fondaparinux (Arixtra) must be reduced if kidney disease is present. (D) A full-strength aspirin can be used solely for deep venous thrombosis prophylaxis. (E) The efficacy of fondaparinux (Arixtra) can be followed by measuring partial thromboplastin time (PTT) levels.

(E) Tracheoesophageal fistula

You are called to see a 4-hour-old neonate in the well-baby nursery who has developed bilious vomiting after taking his first feeding. He was born at 39-week gestation, has not yet passed meconium and has an unremarkable examination. An upper gastrointestinal (GI) series would be the study of choice to rule out which of the following clinical conditions? (A) Ileal atresia (B) Meconium ileus (C) Duodenal web (D) Malrotation (E) Tracheoesophageal fistula

E. Cesarean section The patient is a diabetic with a history of a macrosomic infant; the likelihood of macrosomia in this infant is significant. Instrument delivery is not recommended if macrosomia is suspected. By definition, she has had an arrest of descent of the fetal head and one should be highly suspicious for macrosomia, in which case a c-section is the preferred method of delivery.

You are caring for a 29-yo G3P2 at 39 weeks gestation, who has been laboring for 6 hours. She is a diet-controlled diabetic. Her last child was 9 pounds 8 ounces. She has been completely dilated for 2 hours, and the fetal head is at a plus 2 station, which is unchanged. What is the next most appropriate course of action? A. Begin oxytocin B. Vacuum extraction C. High forcep extraction D. High forcep rotation E. Cesarean section

B. Assessment of dilatation The presence of recurrent late decelerations should raise the suspicion for fetal distress. Vaginal evaluation for change in dilatation or cord prolapse, and to assess the fetal response to stimulation, are the first steps in evaluating the need for intervention.

You are monitoring a 30yo G2P1 at 40 weeks gestation, who is in an active stage of labor and is 6-cm dilated. The FHT has a baseline HR of 140, with 7 to 10 beats of variability. With the last five contractions you have noted late decelerations. What would be the next most appropriate course of action? A. Close observation of FHR tracing B. Assessment of dilatation C. Augment contractions with oxytocin D. Intravenous analgesic E. Surgical intervention

(B) hemolytic-uremic syndrome

You are seeing a 4-year-old child in the emergency department with abdominal pain, and acute onset of bloody diarrhea. Due to a recent outbreak in the community related to a popular fast food restaurant, you suspect E coli 0157:H7. Which of the following is seen most commonly as a complication of Shiga-toxin-producing E coli (formerly known as enterohemorrhagic E coli) diarrhea? (A) meningitis (B) hemolytic-uremic syndrome (C) chronic diarrhea (D) endocarditis (E) pneumonia

(A) hepatitis B, DTaP, HIB, IPV, PCV

You are seeing a 6-month-old for well-child visit and immunizations. Anticipatory guidance is given. What immunizations would you prescribe if the child were up to date at that point? (A) hepatitis B, DTaP, HIB, IPV, PCV (B) hepatitis A, hepatitis B, DTaP, HIB, PCV (C) DTaP, HIB, IPV, PCV, MMR (D) hepatitis A, DTaP, HIB, IPV, PCV (E) DTaP, HIB, MMR, varicella, PCV

(C) Rh incompatibility

You are the attending physician for a newborn infant with hemolytic jaundice. The mother did not receive prenatal care with this pregnancy or her prior pregnancy. The direct Coombs test is positive. The mother's blood type is A- and the baby's blood type is O+. Her first baby did not have hemolytic jaundice. What is the most likely cause of the hemolytic jaundice? (A) ABO incompatibility (B) Toxoplasmosis (C) Rh incompatibility (D) rubella (E) hereditary spherocytosis

(D) mumps

You are working in a clinic in rural Mexico and examine an 8-year-old boy who has a rectal temperature of 100°F (38°C), bilateral tender parotid swelling, and pain when you flex his neck. He has been complaining of a headache. His immunization history is unknown. What is the most likely cause of this child's infection? (A) brucellosis (B) cysticercosis (C) Epstein-Barr virus infection (D) mumps (E) leukemia

c) Harsh systolic crescendo-decrescendo murmur radiating to the carotids

You have been asked to perform a preoperative consultation on a 65-year-old male who will be undergoing a testicular hernia repair. Of the following findings, which is of most concern in predicting a cardiac complication in this patient undergoing noncardiac surgery? a) Age over 60 b) History of myocardial infarction 3.5 years ago c) Harsh systolic crescendo-decrescendo murmur radiating to the carotids d) ECG and subsequent telemetry showing up to five PVCs per minute e) Serum creatinine 2.0 mg/dL

(D) Pyelonephritis

You're evaluating a 65-yo woman who presents with fever and acute LLQ pain. She states that it began last night and won't let up. She says that it began in the back and radiates to the lower left-quadrant area. She denies nausea, vomiting, or diarrhea. She has no history of diverticulosis. Her temp is 38.9°C (102°F). There is LLQ tenderness & left costovertebral tenderness. She admits to dysuria and urinary frequency. The urinalysis is pending. What is the most likely diagnosis? (A) Diverticulitis (B) Volvulus (C) Ovarian torsion (D) Pyelonephritis (E) Ulcerative proctitis

(D) Ampicillin-sulbactam (Unasyn) good choice for intra-abdominal surgeries because it has good Gram-positive, Gram-negative, and anaerobic coverage.

You're preparing a patient to go into surgery for emergent cholecystectomy. The patient presented with a fever of 38.9°C (102°F) and acute right upper-quadrant pain. Ultrasound demonstrates ductal dilatation, thickening of the gallbladder wall, and pericholecystic fluid. The patient is made NPO and started on intravenous fluids. Which antibiotic would be appropriate to administer? (A) Vancomycin (Vancocin) (B) Gentamicin (Garamycin) (C) Metronidazole (Flagyl) (D) Ampicillin-sulbactam (Unasyn) (E) Azithromycin (Zithromax)

C. iritis Iritis is treated with steroid drops (dexamethasone) and miotic drops *homatropine, like atropine (to constrict and fix the pupil to help the pain and open the angle until the iritis is resolved)

Your 27 year-old sister is visiting and requests you to provide refills of dexamethasone and homatropine ophthalmic drops for her. What condition is most likely being treated? A. conjunctivitis B. glaucoma C. iritis D. Herpes keratitis E. blepharitis

(E) a major risk factor for later developmen- tal and behavioral difficulties Failure to thrive is a major risk factor for later behavioral and developmental problems. It is diagnosed by persistent and significant deviation from the growth curves along time. Poverty-stricken children are likely to be affected, at 5%-10% prevalence. *Although this could represent an organic disorder, by far the most common cause of failure to thrive is psychoso- cial: the so called nonorganic failure to thrive.*

he mother of a 6-month-old girl presents for a well-child check. The baby's weight and length are at the 5th percentile, and her head circumference is at the 25th percentile. Which of the following is a true statement regarding failure to thrive in infants? (A) more common among Hispanic children (B) extremely rare in the United States (C) most often caused by an organic problem (D) more common among female infants than male infants (E) a major risk factor for later developmen- tal and behavioral difficulties


Related study sets

Bio Exam AP Psychology (Neurons and Brain)

View Set

3 statement flow through questions

View Set

Anatomy: Chapter 19 - 22 - Quiz Questions

View Set

Empathy: The Foundation of Caring

View Set

Conversion Disorder/Factitious disorder

View Set

Customer Service: Through the Customer's Eyes

View Set

Geometry Definitions Used in Proofs

View Set